0% found this document useful (0 votes)
33 views104 pages

Akt Practice

The MS AKT Practice Paper Exam consists of a 200-item practice exam designed for medical students preparing for the UK Medical School Applied Knowledge Test, reflecting the GMC Content Map. The document includes various clinical scenarios with questions and justifications for the correct answers, covering topics such as urinary symptoms, cardiovascular management, autoimmune disorders, and more. The practice exam is reviewed annually and is intended for personal, educational, and non-commercial use only.

Uploaded by

hahuyhoanghhh41
Copyright
© © All Rights Reserved
We take content rights seriously. If you suspect this is your content, claim it here.
Available Formats
Download as PDF, TXT or read online on Scribd
0% found this document useful (0 votes)
33 views104 pages

Akt Practice

The MS AKT Practice Paper Exam consists of a 200-item practice exam designed for medical students preparing for the UK Medical School Applied Knowledge Test, reflecting the GMC Content Map. The document includes various clinical scenarios with questions and justifications for the correct answers, covering topics such as urinary symptoms, cardiovascular management, autoimmune disorders, and more. The practice exam is reviewed annually and is intended for personal, educational, and non-commercial use only.

Uploaded by

hahuyhoanghhh41
Copyright
© © All Rights Reserved
We take content rights seriously. If you suspect this is your content, claim it here.
Available Formats
Download as PDF, TXT or read online on Scribd
You are on page 1/ 104

MS AKT Practice Paper Exam

Part 1

The MS AKT Exam Board has put together a 200-item practice exam (2 x 100 item
papers) to help medical students prepare for the UK Medical School Applied
Knowledge Test (MS AKT). Blueprinted to the GMC Content Map this exam has
been designed to reflect the style and type of question that students will encounter
when the MS AKT goes live in 2024-25 or 2023-24 for schools that have penultimate
year exams.

The practice exam comes with and without the answer options.

We would like to recognise the contribution of medical schools, and members of the
AKT Exam Board in particular, in producing this exam which we hope students will
find a valuable resource.

Please note this practice exam is reviewed on an annual basis and updated
accordingly. Should you have any questions about the clinical content of the practice
exam please speak to the Assessment Lead in your school in the first instance.

Any redistribution or reproduction of part or all of the contents in any form is


prohibited other than for personal, educational and non-commercial use.

© Medical Schools Council 2024 – reviewed August 2024


Page 1 of 104
1. A 24 year old man has poor urine flow and takes a very long time to empty his
bladder. He has no other urinary symptoms. He has been well previously
apart from one episode of non-gonococcal urethritis 1 year ago.

Which is the most likely diagnosis?

A. Overactive bladder
B. Neurogenic bladder
C. Phimosis
D. Prostatic hypertrophy
E. Urethral stricture
Correct Answer(s): E

Justification for correct answer(s): Based on the symptoms described, the most
likely diagnosis for the 24 year old man is urethral stricture. Urethral stricture
is a condition that occurs when the urethra narrows, which can cause difficulty
in passing urine and a slow urinary stream. This can lead to a feeling of
incomplete emptying of the bladder and a need to strain to empty the bladder
completely. Urethral stricture follows previous urethral inflammation due to
infection. Other possible causes of these symptoms include prostatic
hypertrophy, but this condition is more common in older men, usually over the
age of 50. Overactive bladder and neurogenic bladder can also cause urinary
symptoms, but they typically present with other symptoms such as urgency,
frequency, and incontinence. Phimosis refers to the condition where the
foreskin cannot be retracted from the tip of the penis, and is unlikely to cause
the urinary symptoms described.

© Medical Schools Council 2024 – reviewed August 2024


Page 2 of 104
2. A 67 year old man is found to have an ejection systolic murmur. He is
otherwise well.

His pulse is 72 bpm and BP 128/84 mmHg. His chest is clear.

Investigations:
ECG shows sinus rhythm.
Echocardiography shows aortic stenosis, valve gradient 50 mmHg. Left
ventricular (LV) diastolic dysfunction, LV ejection fraction 45% (>55).

Which is the most appropriate management?

A. Clinical review and echocardiography in 6 months


B. Reassure and discharge
C. Refer for aortic valve replacement
D. Start bisoprolol fumarate and advise review if symptomatic
E. Start lisinopril and advise review if symptomatic
Correct Answer(s): C

Justification for correct answer(s): Aortic stenosis with left ventricular ejection
fraction (LVEF) less than 55% should be referred for consideration of an AVR.
Medications should not be started as these have no effect on the valve disease
progression and may even cause side effects. The patient needs to start the
process of definitive treatment with valve replacement so it is not good
practice or safe to either discharge or review in 6 months.

NICE guideline [NG208] Published: 17 November 2021


https://www.nice.org.uk/guidance/ng208/chapter/recommendations

© Medical Schools Council 2024 – reviewed August 2024


Page 3 of 104
3. A 27 year old woman has muscle weakness which is worse on exercise.
When asked to count from one to 100 her voice progressively becomes
weaker. She has bilateral ptosis.

Which pathophysiological process is most likely to be responsible for this disorder?

A. Autoimmunity
B. Genetic disorder
C. Infarction
D. Malignancy
E. Meningeal infection
Correct Answer(s): A

Justification for correct answer(s): Autoimmunity is the most likely


pathophysiological process responsible for this disorder. The symptoms
described are consistent with myasthenia gravis, an autoimmune disorder that
affects neuromuscular transmission. The muscle weakness worsens with
exercise or prolonged use, and the patient also has bilateral ptosis, which is a
common feature of myasthenia gravis. The progressive weakness of voice
during counting is a characteristic finding known as the "fatigue test".
Autoimmune antibodies target the acetylcholine receptors at the
neuromuscular junction, resulting in impaired transmission of nerve impulses
to the muscle.

© Medical Schools Council 2024 – reviewed August 2024


Page 4 of 104
4. A 60 year old man has 6 months of dry cough and increasing shortness of
breath on effort. He was previously fit and well, and is a non-smoker.

His temperature is 36.8°C, pulse rate 60 bpm and oxygen saturation 89%
breathing air.

He has finger clubbing. Cardiac examination is normal, and chest examination


reveals bibasal crepitations.

Which is the most likely diagnosis?

A. Bronchiectasis
B. Extrinsic allergic alveolitis
C. Idiopathic pulmonary fibrosis
D. Lung carcinoma
E. Pulmonary tuberculosis
Correct Answer(s): C

Justification for correct answer(s): All of the answers are possible causes of
cough, finger clubbing and increasing breathlessness but Idiopathic
Pulmonary Fibrosis (IPF) is most likely.

The dry cough, increasing shortness of breath on exertion, hypoxia, finger


clubbing and bibasal crepitations are all in keeping with IPF. Lung cancer is
less likely as the patient is a non-smoker and the signs are bilateral in the
chest. Extrinsic allergic alveolitis is a possible correct answer but it is less
common than IPF and it is usually associated with a history of exposure to a
specific antigen. Pulmonary TB is less likely as there is no fever, the cough is
dry and the signs are bibasal.

© Medical Schools Council 2024 – reviewed August 2024


Page 5 of 104
5. A 46 year old man has pain in his left leg and tingling in his left big toe. He
developed severe lower back pain 1 week ago and he is unable to walk on his
left heel.

There is loss of pinprick perception over the left great toe.

Which nerve root is the most likely to have been affected?

A. L1
B. L3
C. L5
D. S1
E. S2
Correct Answer(s): C

Justification for correct answer(s): L5 is the most likely nerve root to have been
affected. The patient has a combination of lower back pain, pain in the left leg,
and tingling in the left big toe, which are consistent with the dermatomal
distribution of the L5 nerve root. The inability to walk on the left heel suggests
a left-sided foot drop, and so is also consistent with L5 nerve root dysfunction.
The loss of pinprick perception over the left great toe also suggests
involvement of the L5 dermatome.

© Medical Schools Council 2024 – reviewed August 2024


Page 6 of 104
6. A 65 year old man attends the anticoagulant clinic. He has had a metal mitral
valve replacement and atrial fibrillation. He takes warfarin 7 mg daily.

His pulse rate is 70 bpm, irregularly irregular, with a mechanical second heart
sound. There are no signs of bleeding.

His INR is 5.1.

Which is the most appropriate next step in management?

A. Continue warfarin at lower dose


B. Continue warfarin at same dose
C. Give vitamin K intravenously
D. Give vitamin K orally
E. Withhold warfarin for 2 days then restart at lower dose
Correct Answer(s): E

Justification for correct answer(s): It is important to maintain a therapeutic INR to


avoid thromboembolism from both the metallic valve and atrial fibrillation.
However, we also have to manage the risk of bleeding. With the INR of 5.1 there
is a risk of bleeding which can be catastrophic if affecting vital organs such as
the brain. In this situation with no bleeding, it is safe to withhold the warfarin
for 2 days to allow it to reduce naturally and then perhaps start at a lower dose
of warfarin than before. Not stopping warfarin runs the risk that the INR fails to
fall with a risk of bleeding (A and B wrong). Reducing the INR too quickly with
vitamin K (IV or oral) in the absence of bleeding, can lead to the development
of thromboembolisms occurring and also challenges in maintaining future
therapeutic levels (requirement for SC heparin).

© Medical Schools Council 2024 – reviewed August 2024


Page 7 of 104
7. A 52 year old man has three days of severe epigastric pain, radiating to his
back, but no chest pain. He has vomited several times. He was previously
well. He drinks approximately 60 units of alcohol a week and smokes 20
cigarettes per day.

There is epigastric tenderness but his abdomen is not distended, and bowel
sounds are present.

Which test would confirm the most likely diagnosis?

A. Abdominal X-ray
B. Gastroduodenoscopy
C. Serum alkaline phosphatase concentration
D. Serum amylase concentration
E. Ultrasound scan of abdomen
Correct Answer(s): D

Justification for correct answer(s): Serum amylase concentration would confirm


the most likely diagnosis in this case. The patient presents with severe
epigastric pain, radiating to his back, and vomiting, which are suggestive of
acute pancreatitis. The presence of epigastric tenderness but not distended
abdomen and normal bowel sounds are also consistent with this diagnosis.
Serum amylase concentration is an important diagnostic test for acute
pancreatitis. Elevated serum amylase levels occur early in the course of the
disease and can be measured within hours of symptom onset. Abdominal X-
ray, gastroduodenoscopy, serum alkaline phosphatase concentration, and
ultrasound scan of the abdomen may be useful in certain cases, but they are
not as specific for diagnosing acute pancreatitis as serum amylase
concentration.

© Medical Schools Council 2024 – reviewed August 2024


Page 8 of 104
8. A 24 year old man attends the Emergency Department after 2 days of
vomiting. He has type 1 diabetes. He is drowsy but maintaining his airway.

His pulse rate is 100 bpm, BP 90/60 mmHg, respiratory rate 30 breaths per
minute and oxygen saturation 96% breathing air.

Investigations:
Blood capillary glucose 32 mmol/L
Blood capillary ketones 6.2 mmol/L (<0.6)
Venous pH 7.15 (7.35–7.45)

Which is the most appropriate initial treatment?

A. Intravenous 0.9% sodium chloride


B. Intravenous 1.26% sodium bicarbonate
C. Intravenous antibiotics
D. Intravenous insulin
E. Subcutaneous insulin
Correct Answer(s): A

Justification for correct answer(s): This patient has diabetic ketoacidosis (DKA).
The first step in treatment is intravenous 0.9% sodium chloride to correct
dehydration and hyperosmolality. IV insulin will be required but follows initial
fluid prescription.

© Medical Schools Council 2024 – reviewed August 2024


Page 9 of 104
9. A 55 year old man is rescued from a collapsed building where he has been
trapped for 12 hours without water.His temperature is 35.6°C, pulse rate 100
bpm and BP 90/42 mmHg. His JVP is not visible. His abdomen is non tender.

Investigations:
Haemoglobin 168 g/L (130–175)
Sodium 148 mmol/L (135–146)
Potassium 6.0 mmol/L (3.5–5.3)
Urea 25.1 mmol/L (2.5–7.8)
Creatinine 184 μmol/L (60–120)
Creatine kinase 840 U/L (25–200)

Which is the most likely cause of this biochemical picture?

A. Bladder outflow obstruction


B. Direct renal trauma
C. Hypovolaemia
D. Rhabdomyolysis
E. Sepsis
Correct Answer(s): C

Justification for correct answer(s): The most likely cause is acute kidney injury
due to hypovolaemia. The observations of tachycardia and hypotension fit this.
There are no signs of sepsis or reason why this has developed. The creatine
kinase is only minimally elevated and would normally be >10,000 in cases of
rhabdomyolysis.

© Medical Schools Council 2024 – reviewed August 2024


Page 10 of 104
10. A 24 year old woman has diarrhoea. She is HIV positive and has been
working in Namibia.

Investigation:
Faeces microscopy (following modified Ziehl–Neelsen stain): protozoa

Which is the most likely causative organism?

A. Acanthamoeba
B. Cryptosporidium parvum
C. Entamoeba coli
D. Plasmodium falciparum
E. Schistosoma mansoni
Correct Answer(s): B

Justification for correct answer(s): Cryptosporidium parvum is the most likely


causative organism for diarrhoea in an HIV positive patient working in Namibia.
Cryptosporidium is a protozoan parasite that can cause diarrhoea in
immunocompromised patients, including those with HIV. It is commonly found
in contaminated water sources and is a significant cause of diarrhoeal disease
in developing countries. Acanthamoeba, Entamoeba coli, Plasmodium
falciparum, and Schistosoma mansoni can also cause various diseases, but
are less likely to be the cause of diarrhoea in this patient given the clinical
context.

© Medical Schools Council 2024 – reviewed August 2024


Page 11 of 104
11. A 67 year old man has difficulty chewing and speaking. He underwent carotid
surgery 2 days ago.

His tongue deviates to the right when he is asked to protrude it.

Which nerve has been damaged?

A. Left glossopharyngeal nerve


B. Left hypoglossal nerve
C. Left vagus nerve
D. Right glossopharyngeal nerve
E. Right hypoglossal nerve
Correct Answer(s): E

Justification for correct answer(s): The hypoglossal nerve is responsible for motor
function of the tongue, including protrusion and side-to-side movements.
Damage to the hypoglossal nerve on one side will cause the tongue to deviate
towards the affected side (the stronger left side will push it to the right). In this
case, the patient had carotid surgery on the left side, so the right hypoglossal
nerve is likely to have been damaged.

© Medical Schools Council 2024 – reviewed August 2024


Page 12 of 104
12. An 18 year old woman has had 3 years of intermittent zig-zagging and
flashing lights in both eyes associated with headache. These episodes occur
2-3 times per month and last approximately half an hour. The symptoms are
associated with nausea and vomiting. Her vision is affected at the time of
each episode but returns to normal afterwards.

Which is the most likely diagnosis?

A. Acute glaucoma
B. Migraine
C. Occipital lobe epilepsy
D. Retinal detachment
E. Tension-type headache
Correct Answer(s): B

Justification for correct answer(s): The most likely diagnosis is B. Migraine. The
classic visual symptoms of zig-zagging and flashing lights, headache, nausea,
and vomiting are typical features of migraine with aura. The fact that the
patient’s vision returns to normal after each episode is also consistent with
migraine. Acute glaucoma and retinal detachment may present with sudden
onset of symptoms, but these are a ‘one off’ and persist, and require urgent
ophthalmologic assessment. Occipital lobe epilepsy is a possibility but less
likely since zig-zags almost always point to migraine rather than epilepsy.
Tension-type headache does not typically have visual symptoms.

© Medical Schools Council 2024 – reviewed August 2024


Page 13 of 104
13. A 75 year old man has had 3 days of intermittent headaches, blurred vision
and vomiting. For the past 24 hours he has had a severe left sided headache
and eye pain, accompanied by blurred vision and vomiting. His left eye is red
and the left pupil is dilated.

Which investigation is most likely to confirm the diagnosis?

A. CT scan of head
B. Erythrocyte sedimentation rate
C. Fluorescein staining of the cornea
D. Measurement of intraocular pressure
E. MR scan of head
Correct Answer(s): D

Justification for correct answer(s): Based on the symptoms and signs described,
the most likely diagnosis is acute angle-closure glaucoma, which is a medical
emergency that requires prompt diagnosis and treatment to prevent vision
loss. Therefore, the investigation that is most likely to confirm the diagnosis is
measurement of intraocular pressure. A high intraocular pressure is
characteristic of acute angle-closure glaucoma, although other investigations
such as a CT or MR scan of the head may be performed to rule out other
causes of the symptoms.

© Medical Schools Council 2024 – reviewed August 2024


Page 14 of 104
14. A 72 year old woman has had 4 months of progressive difficulty walking. She
describes numbness and tingling in her feet and has fallen on several
occasions.

She has normal tone of her lower limbs, moderate weakness of ankle
dorsiflexion and plantar flexion, normal knee jerks, but absent ankle jerks and
extensor plantars. Romberg's test is positive. She has reduced vibration
sense, and joint position sense is impaired up to the ankle joints. Temperature
and pinprick sensations are normal.

Which investigation is most likely to confirm the diagnosis?

A. HbA 1c
B. Serum folate
C. Serum protein electrophoresis
D. Serum vitamin B 12
E. Serum vitamin D
Correct Answer(s): D

Justification for correct answer(s): The clinical picture is consistent with subacute
combined degeneration of the cord, giving a mixture of upper motor neurone
(extensor plantars) and lower motor neurone (absent ankle jerks) features. The
sensory ataxia (positive Romberg’s test and absent position sense in the
ankles) is most likely due to dorsal column dysfunction from vitamin
B12 deficiency, and this can be confirmed by serum vitamin B12 measurement.
Vitamin B12 deficiency of this severity is usually caused by pernicious anaemia.

© Medical Schools Council 2024 – reviewed August 2024


Page 15 of 104
15. A 34 year old woman has a recurrent itchy rash which lasts for several hours
before resolving (see image). She has not identified any triggers. She is
systemically well. She is a firefighter and says that she does not want any
treatments that may affect her level of alertness.

Which is the most appropriate treatment to control her symptoms?

A. Oral chlorphenamine maleate


B. Oral loratadine
C. Oral prednisolone
D. Topical aqueous cream
E. Topical hydrocortisone
Correct Answer(s): B

Justification for correct answer(s): The image shows urticarial weals. Initial
treatment for this should be a non-sedating H1-antihistamine. The correct
answer is thus B (oral loratadine). Chlorphenamine maleate is a sedating
antihistamine, which is more likely to cause adverse effects; this patient also
specifically requested treatment that would not affect her level of alertness.
Prednisolone is effective for severe, acute urticaria but should not be used
first-line. Aqueous cream is a soap substitute and has no role in the

© Medical Schools Council 2024 – reviewed August 2024


Page 16 of 104
management of urticaria. Topical corticosteroids are ineffective for urticaria so
hydrocortisone is not indicated here.

© Medical Schools Council 2024 – reviewed August 2024


Page 17 of 104
16. A 29 year old woman has 2 days of marked loss of vision and acute pain in
her left eye. The pain is worse when she changes her gaze direction.

Her eyes appear normal on general inspection. Her vision is 'count fingers
only' in the affected eye. The swinging flashlight test shows that the left pupil
dilates when a bright light is moved from the right eye to the left eye. The optic
discs are normal on fundoscopy.

Which is the most likely diagnosis?

A. Acute closed angle glaucoma


B. Giant cell arteritis
C. Idiopathic intracranial hypertension
D. Migraine with aura
E. Retrobulbar optic neuritis
Correct Answer(s): E

Justification for correct answer(s): The most likely diagnosis in this scenario is
retrobulbar optic neuritis. The acute onset of eye pain and marked loss of
vision, along with the presence of relative afferent pupillary defect (RAPD) on
swinging flashlight test, are suggestive of optic neuritis. The absence of optic
disc swelling on fundoscopy suggests a retrobulbar lesion. Acute closed angle
glaucoma also presents with acute eye pain, but it is typically associated with
other features such as vomiting, headaches and a red eye with a dilated pupil
accompanied by a high intraocular pressure. Giant cell arteritis can also cause
acute visual loss, but it is more commonly seen in older patients and is often
associated with systemic symptoms such as headache, jaw claudication, and
malaise. Idiopathic intracranial hypertension can cause vision loss and
headache, but it typically does not cause pain with eye movements. Migraine
with aura can cause visual disturbances, but it is typically not associated with
pain, and the presence of RAPD suggests a neuro-ophthalmic rather than a
primary headache disorder.

© Medical Schools Council 2024 – reviewed August 2024


Page 18 of 104
17. A 45 year old woman develops an intensely painful eruption around her right
eye. The illness started with pain 5 days previously, followed by the
appearance of a few vesicles, which has now developed into the rash (see
image). She has no significant medical history.

Treatment is started.

Which is the most likely long-term outcome?

A. Complete resolution with no sequelae


B. Corneal ulceration
C. Extensive scarring of the right temple
D. Partial ptosis
E. Reduced visual acuity
Correct Answer(s): A

Justification for correct answer(s): The clinical presentation described in this


scenario is consistent with herpes zoster ophthalmicus, which is caused by
reactivation of the varicella-zoster virus in the ophthalmic division of the
trigeminal nerve. Ocular involvement occurs in approximately 50% of patients
and some of these can experience a range of complications. However, in the
majority of cases there is complete resolution with no sequelae.

© Medical Schools Council 2024 – reviewed August 2024


Page 19 of 104
18. A 48 year old woman has rheumatoid arthritis. She takes regular paracetamol
and has no drug allergies. She is due to commence methotrexate weekly.

Which additional treatment should be prescribed?

A. Calcium carbonate
B. Folic acid
C. Pyridoxine hydrochloride
D. Thiamine
E. Vitamin D
Correct Answer(s): B

Justification for correct answer(s): Folic acid is recommended as an additional


treatment for patients taking methotrexate to reduce the risk of adverse effects.
It can be used for this purpose once a week, or daily but omitted on the day of
the (weekly) methotrexate dose. Calcium carbonate, pyridoxine hydrochloride,
thiamine, and vitamin D are not routinely prescribed as additional treatments
for methotrexate. Pyridoxine hydrochloride is usually prescribed alongside
isoniazid in the treatment of tuberculosis.

© Medical Schools Council 2024 – reviewed August 2024


Page 20 of 104
19. A 74 year old woman has 6 months of progressive weakness of her right leg
and 3 months of a similar problem on the left, resulting in several falls. She
has also noticed difficulty using her hands and can no longer fasten the
buttons on her clothes.

There is wasting of both legs and the hands, particularly the thenar
eminences. There is fasciculation in her right quadriceps. Tone is increased in
both legs, with brisk reflexes.

Which is the most likely diagnosis?

A. Chronic inflammatory demyelinating polyneuropathy


B. Motor neurone disease
C. Multiple sclerosis
D. Myasthenia gravis
E. Polymyositis
Correct Answer(s): B

Justification for correct answer(s): The most likely diagnosis is motor neurone
disease, which is characterised by progressive weakness and wasting of
muscles due to degeneration of motor neurones in the brain and spinal cord.
The combination of both lower motor neurone signs (fasciculation) and upper
motor neurone signs (brisk reflexes) in the same limb are particularly
characteristic of motor neurone disease. Multiple sclerosis is purely upper
motor neurone and the options listed (chronic inflammatory demyelinating
polyneuropathy, myasthenia gravis, and polymyositis) are purely lower motor
neurone conditions.

© Medical Schools Council 2024 – reviewed August 2024


Page 21 of 104
20. A 61 year old man has had 2 months of ankle swelling. He has hypertension
and a 30 year history of seronegative polyarthritis. His medication includes
ramipril, sulfasalzine, hydroxychloroquine sulfate and diclofenac.

His BP is 156/90 mmHg. He has pitting oedema to mid thigh and signs of
chronic deforming polyarthropathy in his hands, but no joint tenderness. His
optic fundi show silver wiring and arteriovenous nipping. Urinalysis: protein
4+, no other abnormalities.

Investigations:
Sodium 133 mmol/L (135–146)
Potassium 5.4 mmol/L (3.5–5.3)
Urea 9.0 mmol/L (2.5–7.8)
Creatinine 119 µmol/L (60–120)
Albumin 21 g/L (35–50)
CRP 43 mg/L (<5)
Urinary protein:creatinine ratio 1100 mg/mmol (<30)

Which is the most appropriate initial treatment?

A. Candesartan cilexetil
B. Furosemide
C. Indapamide
D. Prednisolone
E. Prednisolone and cyclophosphamide
Correct Answer(s): B

Justification for correct answer(s): Based on the clinical presentation and


investigations, the most likely diagnosis is nephrotic syndrome, possibly
secondary to the patient’s long-standing polyarthritis. The appropriate initial
treatment would be to start a furosemide to reduce the patient’s ankle swelling
and to refer the patient to a specialist for further investigation and
management of the underlying cause. Furosemide is a loop diuretic that acts
on the ascending limb of the loop of Henle to increase sodium and water
excretion, which can reduce oedema. It is a commonly used diuretic in the
management of nephrotic syndrome.

© Medical Schools Council 2024 – reviewed August 2024


Page 22 of 104
21. A 75 year old woman has had 5 months of a 2 cm red plaque on her leg.

Investigation:
Skin biopsy: Bowen's disease

Which is the most appropriate topical treatment?

A. 5-fluorouracil (Efudix ® ) cream


B. Betamethasone valerate (Betnovate ® ) cream
C. Diclofenac (Solaraze ® ) gel
D. Isotretinoin gel
E. Salicylic acid gel
Correct Answer(s): A

Justification for correct answer(s): The most appropriate topical treatment for
Bowen’s disease, a type of squamous cell carcinoma in situ 5-fluorouracil
(Efudix®) cream. This is a form of topical cytotoxic chemotherapy which is
used to treat both Bowen’s disease and actinic keratosis. It is typically applied
to the affected area once or twice a day for 2-4 weeks. An inflammatory
reaction, which can be severe, should be expected.

Topical corticosteroids such as betamethasone valerate have no effect on


Bowen’s disease. Diclofenac can be used in the treatment of actinic keratosis
but is not indicated for Bowen’s disease. Isotretinoin gel is a retinoid used in
the treatment of acne. Salicylic acid is a keratolytic agent, which is used in the
management of hyperkeratotic lesions such as viral warts and sometimes
actinic keratoses. Whilst it might reduce hyperkeratosis in Bowen’s, it will not
treat the underlying dysplasia effectively.

© Medical Schools Council 2024 – reviewed August 2024


Page 23 of 104
22. A 32 year old woman has had palpitations and hot flushes for 4 weeks. She
has noticed a painless swelling in her neck over the same time and her weight
has decreased by 2 kg. She gave birth 4 months ago after a normal
pregnancy. She is not breastfeeding.

Her pulse rate is 120 bpm and BP 140/90 mmHg. She is tremulous and
restless. She has a large smooth non-tender goitre.

Investigations:
Free T4 35.6 pmol/L (9–25)
Free T3 10.8 pmol/L (4.0–7.2)
TSH <0.01 mU/L (0.3–4.2)
Thyroid peroxidase antibodies >1600 IU/L (<50)
Thyroid stimulating antibodies <1.0 IU/L (<1.75)

Which is the most appropriate initial treatment?

A. Carbimazole
B. Propranolol
C. Propylthiouracil
D. Thyroidectomy
E. Thyrotropin alfa
Correct Answer(s): B

Justification for correct answer(s): The patient’s presentation and investigations


are consistent with hyperthyroidism and a diagnosis of postpartum thyroiditis.
Given her symptoms of palpitations, hot flushes, tremulousness, and a high
pulse rate, the most appropriate initial treatment is option propranolol. It works
by blocking the effects of thyroid hormones on the heart and peripheral
tissues. Propranolol can be started immediately to control the patient’s
symptoms while further investigations and management are initiated.

© Medical Schools Council 2024 – reviewed August 2024


Page 24 of 104
23. An 80 year old man has an ulcer over the left heel and reduced mobility. He
has a loss of appetite. He has type 2 diabetes mellitus and has previously had
a myocardial infarction.

The ulcer is 3 cm in diameter and deeply penetrating. Sensory testing shows


reduced vibration sense but normal sensation to light touch.

His Doppler ratio (ankle brachial pressure index) on the left is 0.68 and on the
right is 0.98 (normal value 1.00).

Which is the most likely mechanism of his ulcer?

A. Arterial
B. Neuropathic
C. Nutritional
D. Vasculitic
E. Venous
Correct Answer(s): A

Justification for correct answer(s): The ulcer description is arterial. Given the APB
of 0.68 and a history of previous MI to support this diagnosis. Normal
sensation rules out neuropathic. Nutritional, venous and vasculitic ulcers have
different characteristics.

© Medical Schools Council 2024 – reviewed August 2024


Page 25 of 104
24. A 73 year old man is in hospital with a chest infection. He has several
episodes of confusion, anxiety and aggression, during which he attempts to
leave the hospital. He is convinced he is being 'spied on' by the doctors and
nurses and insists that 'cameras have been installed in my room'. These
episodes alternate with periods of marked lethargy, which become more
pronounced towards the evening.

His temperature is 37.8°C, pulse rate 100 bpm, BP 110/73 mmHg and
respiratory rate 12 breaths per minute.

Which is the most likely diagnosis?

A. Alzheimer's dementia
B. Bipolar disorder
C. Delirium
D. Lewy body dementia
E. Schizophrenia
Correct Answer(s): C

Justification for correct answer(s): Delirium is the most likely diagnosis given the
patient’s acute onset of confusion, fluctuating level of consciousness,
perceptual disturbances, and physical illness (chest infection) as a
precipitating factor. Delirium is a common acute neuropsychiatric disorder
among hospitalised elderly patients and can be caused by a variety of factors
such as infection, medication side effects and metabolic derangements.
Alzheimer’s dementia and Lewy body dementia are chronic neurodegenerative
disorders characterized by progressive cognitive decline and are not typically
associated with acute changes in mental status. Bipolar disorder and
schizophrenia are chronic psychiatric disorders that may cause psychosis and
delusions but are not typically associated with the acute onset of confusion
seen in delirium.

© Medical Schools Council 2024 – reviewed August 2024


Page 26 of 104
25. A 78 year old woman is admitted to the surgical unit with a suspected
vesicocolic fistula. She has hypertension, type 2 diabetes mellitus and angina.
She takes amlodipine, metformin, gliclazide, simvastatin and bisoprolol.Her
serum creatinine is 120 µmol/L (60–120).

The consultant surgeon requests a CT scan of abdomen with contrast.

Which medication should be stopped before her CT scan?

A. Amlodipine
B. Bisoprolol
C. Gliclazide
D. Metformin
E. Simvastatin
Correct Answer(s): D

Justification for correct answer(s): Metformin should be stopped before a CT scan


with contrast as it can increase the risk of contrast-induced nephropathy. The
risk is greater in patients with impaired renal function like the patient in this
scenario. The other medications do not need to be stopped before the CT scan.

© Medical Schools Council 2024 – reviewed August 2024


Page 27 of 104
26. A 50 year old man has a 3 month history of right loin pain and weight loss. For
the past 20 years, he has smoked ten cigarettes per day.

His temperature is 37.4°C, pulse rate is 72 bpm and BP is 142/74 mmHg.

Investigations:
Haemoglobin 11.2 g/L (130–175)
Platelets 340 × 109/L (150–400)
White cell count 10.1 × 109/L (4.0–11.0)
Urinalysis blood 3+

Which is the most likely diagnosis?

A. Benign prostatic hypertrophy


B. Pyelonephritis
C. Renal calculus
D. Renal cancer
E. Urinary tract infection
Correct Answer(s): D

Justification for correct answer(s): Given the patient’s history of weight loss, loin
pain, and smoking, as well as the presence of blood in the urine, the most
likely diagnosis is renal cancer. Other conditions, such as benign prostatic
hypertrophy, pyelonephritis, renal calculus, and urinary tract infection, may
also present with similar symptoms but are less likely given the patient’s
history and laboratory findings. Further imaging studies, such as a CT scan or
ultrasound, would be needed to confirm the diagnosis.

© Medical Schools Council 2024 – reviewed August 2024


Page 28 of 104
27. A 74 year old man has been increasingly unwell with progressive thirst and
nausea for 2 weeks. He initially described needing to pass urine more
frequently than usual, but now he has not passed urine for 24 hours. He has
type 2 diabetes.

He is dehydrated.

Investigations:
Sodium 149 mmol/L (135–146)
Potassium 5.2 mmol/L (3.5–5.3)
Chloride 101 mmol/L (95–106)
Urea 15.4 mmol/L (2.5–7.8)
Creatinine 208 µmol/L (60–120)
Fasting glucose 41.7 mmol/L (3.0–6.0)

Which is the calculated serum osmolality?

A. 206.1 mmol/L
B. 255.2 mmol/L
C. 312.3 mmol/L
D. 355.1 mmol/L
E. Impossible to calculate, more information needed
Correct Answer(s): D

Justification for correct answer(s):

Serum osmolality is 2 x(Na) + Urea + glucose


The calculation is 298 + 15.4 + 41.6 = 355

© Medical Schools Council 2024 – reviewed August 2024


Page 29 of 104
28. A 65 year old man receives a renal transplant. He is transferred back to the
ward after four hours in recovery.

His pulse is 106 bpm regular, BP 110/70 mmHg and respiratory rate 18
breaths per minute. His chest is clear on auscultation. His urine output has
been 15–20 mL per hour while in recovery. Drain output has been 120 mL
since surgery.

Investigations:
Haemoglobin 90 g/L (130–175) (preoperative level 103 g/L)
Sodium 142 mmol/L (135–146)
Potassium 5.8 mmol/L (3.5–5.3)
Urea 31.9 mmol/L (2.5–7.8)
Creatinine 590 µmol/L (60–120)

Which is the next most appropriate management step?

A. Blood transfusion
B. Fluid challenge
C. Furosemide
D. Haemofiltration
E. Insulin and dextrose infusion
Correct Answer(s): B

Justification for correct answer(s): The patient has a low urine output, with
tachycardia and relative hypotension shortly after a renal transplant. In this
early post-operative phase the most likely cause is hypovolaemia so the most
appropriate intervention would be to administer a fluid challenge.

© Medical Schools Council 2024 – reviewed August 2024


Page 30 of 104
29. A 70 year old man has a sharp stabbing pain in his jaw and cheek that lasts
for seconds. He reports that the pain is triggered when brushing his teeth,
cold wind and touching his face.

Which is the most appropriate treatment?

A. Carbamazepine
B. Indometacin
C. Morphine
D. Prednisolone
E. Pregabalin
Correct Answer(s): A

Justification for correct answer(s): The symptoms described suggest the


diagnosis of trigeminal neuralgia. Carbamazepine is the first-line treatment for
this condition and is therefore the most appropriate option among the choices
given.

Clinical guideline [CG173] Published: 20 November 2013 Last updated: 22


September 2020
https://www.nice.org.uk/guidance/cg173/chapter/Recommendations

© Medical Schools Council 2024 – reviewed August 2024


Page 31 of 104
30. A 64 year old man has developed a tremor in both arms over the last 6
months. It is worse on the right. He also reports difficulty sleeping due to
restlessness.

He appears emotionally flat and has a tremor at rest that is alleviated on


movement.

Which neurotransmitter is most likely to be deficient?

A. Acetylcholine
B. Dopamine
C. Glycine
D. Norepinephrine (noradrenaline)
E. Serotonin
Correct Answer(s): B

Justification for correct answer(s): The diagnosis is Parkinso’s disease and hence
dopamine is most likely to be deficient. The presence of an asymmetric resting
tremor that is alleviated on movement is a characteristic feature of Parkinson’s
disease. The restless sleep implies probable associated REM sleep behaviour
disorder.

© Medical Schools Council 2024 – reviewed August 2024


Page 32 of 104
31. An 85 year old man is admitted from a nursing home with a spreading cellulitis
originating from an ulcer over his right ankle.

He is mildly confused. His temperature is 39.5°C, pulse rate 96 bpm and BP


114/60 mmHg.

Cultures taken from the ulcer and blood have grown MRSA.

Which is the most appropriate initial antibiotic treatment?

A. Co-amoxiclav
B. Flucloxacillin
C. Meropenem
D. Piperacillin with tazobactam
E. Vancomycin
Correct Answer(s): E

Justification for correct answer(s): Vancomycin would be the most appropriate


initial antibiotic treatment in this case of MRSA cellulitis. Vancomycin is a
glycopeptide antibiotic that is active against MRSA and other Gram-positive
bacteria. It is the drug of choice for treating serious MRSA infections, such as
cellulitis, when the strain is known or suspected to be resistant to beta-lactam
antibiotics like flucloxacillin or co-amoxiclav. Piperacillin with tazobactam and
meropenem are broad-spectrum antibiotics that may be used as an alternative
if the patient has a severe penicillin allergy or if the infection is suspected to be
caused by Gram-negative bacteria as well. However, they are not specific for
MRSA and should be used judiciously to avoid the development of antibiotic
resistance.

NICE guideline [NG141] Published: 27 September 2019


https://www.nice.org.uk/guidance/ng141/chapter/Recommendations

© Medical Schools Council 2024 – reviewed August 2024


Page 33 of 104
32. A 75 year old man is found collapsed at home and is brought to the
Emergency Department. He has right sided weakness and reduced
consciousness. He has type 2 diabetes mellitus, atrial fibrillation and
hypertension. He takes warfarin.

Investigations:
INR 4.6 (<1.4)

CT scan of the head shows a large intracranial haemorrhage.

He is given intravenous vitamin K.

Which is the most appropriate next additional treatment?

A. Cryoprecipitate
B. Fresh frozen plasma
C. Fibrinogen concentrate
D. No additional treatment needed
E. Prothrombin complex concentrate
Correct Answer(s): E

Justification for correct answer(s): The patient has an INR of 4.6, which is
significantly elevated, suggesting an excessive anticoagulant effect of
warfarin. The CT scan shows a large intracranial haemorrhage, which is a life-
threatening complication that requires urgent management. The administration
of vitamin K will help to reverse the anticoagulant effect of warfarin, but this
will take several hours to take effect. In the meantime, the patient is at risk of
ongoing bleeding, and so requires further treatment. Prothrombin complex
concentrate (PCC) is the most appropriate next additional treatment in this
situation. PCC is a concentrated source of clotting factors that can be used to
rapidly reverse the anticoagulant effect of warfarin and restore haemostasis. It
is more effective than fresh frozen plasma or cryoprecipitate and has a lower
risk of complications. Fibrinogen concentrate is not indicated in this situation
as there is no evidence of fibrinogen deficiency.

© Medical Schools Council 2024 – reviewed August 2024


Page 34 of 104
33. A 76 year old woman has no energy and reports excessive tiredness for 3
weeks. She has lost 5 kg in weight over the past 3 months. She drinks 30
units of alcohol per week.

She is thin and jaundiced. Her temperature is 37.2°C. She has a palpable
epigastric mass and 4 cm liver edge.

Which is the most likely diagnosis?

A. Cholangiocarcinoma
B. Cholecystitis
C. Cirrhosis of the liver
D. Hepatocellular carcinoma
E. Pancreatic adenocarcinoma
Correct Answer(s): E

Justification for correct answer(s): The most likely diagnosis given the
presentation of the patient is pancreatic adenocarcinoma. The symptoms of
excessive tiredness, weight loss, jaundice, and palpable mass in the epigastric
area along with a history of alcohol use make pancreatic adenocarcinoma the
most probable diagnosis. The presence of a 4cm liver edge also indicates liver
metastasis.

© Medical Schools Council 2024 – reviewed August 2024


Page 35 of 104
34. A 76 year old woman with hypertension is taking amlodipine 10 mg daily.

A 24 hour BP measurement shows a mean BP of 168/90 mmHg.

Investigations:
Sodium 135 mmol/L (135–146)
Potassium 4.0 mmol/L (3.5–5.3)
Urea 7 mmol/L (2.5–7.8)
Creatinine 100 μmol/L (60–120)
eGFR 68 mL/min/1.73 m2(>60)

Urinary albumin : creatinine ratio 50 mg/mmol (<3.5)

Which class of antihypertensive should be added?

A. ACE inhibitor
B. Alpha blocker
C. Beta blocker
D. Loop diuretic
E. Thiazide-like diuretic
Correct Answer(s): A

Justification for correct answer(s): Ace inhibitors are the most effective
medication to treat albuminuria to delay progression to end stage renal
disease and reduces cardiovascular risk. NICE suggest that ACE inhibitors or
AR2B medications should be first choice in this situation with an ACR >30
mg/mmol in a patient with hypertension. There is no evidence for the other
medications to reduce proteinuria and thus CVS risk.

NICE guideline [NG136] Published: 28 August 2019 Last updated: 21


November 2023
https://www.nice.org.uk/guidance/ng136/chapter/recommendations

© Medical Schools Council 2024 – reviewed August 2024


Page 36 of 104
35. A 70 year old man is an inpatient on the cardiology ward. He has worsening
breathlessness that woke him up last night.

His pulse is 99 bpm, BP 160/100 mmHg and respiratory rate 20 breaths per
minute. Auscultation of the chest reveals bibasal crepitations, and there is
dullness to percussion of both bases.

Chest X-ray shows small bilateral pleural effusions with upper lobe blood
vessel diversion.

Which is the most appropriate diagnostic investigation?

A. Coronary angiography
B. CT pulmonary angiography
C. ECG
D. Echocardiography
E. Serum D dimer
Correct Answer(s): D

Justification for correct answer(s): Echocardiography would be the most


appropriate diagnostic investigation for this patient with worsening
breathlessness and signs of heart failure on examination and chest X-ray.
Echocardiography can provide information on cardiac function and identify
possible causes of heart failure such as valvular disease, cardiomyopathy or
pericardial effusion. Coronary angiography would be indicated if there is
suspicion of underlying coronary artery disease, but the presentation in this
case suggests heart failure rather than acute coronary syndrome. CT
pulmonary angiography and serum D-dimer would be indicated if there is
suspicion of pulmonary embolism, but the presence of bilateral pleural
effusions and upper lobe blood vessel diversion on chest X-ray suggests a
cardiac cause for the symptoms. ECG may show evidence of underlying
cardiac disease, but echocardiography would be a more appropriate
investigation to assess cardiac function.

© Medical Schools Council 2024 – reviewed August 2024


Page 37 of 104
36. An 80 year old man has sudden onset of loss of vision in his right eye. He has
hypertension and a previous stroke.

His visual acuity is hand movements only in the right eye and 6/9 in left eye.
The right eye has an afferent pupillary defect; left eye pupil responses are
normal. On fundoscopy there is a red spot at the right macula.

Which is the most likely diagnosis?

A. Anterior ischaemic optic neuropathy


B. Branch retinal vein occlusion
C. Central retinal artery occlusion
D. Macular degeneration
E. Retinal detachment
Correct Answer(s): C

Justification for correct answer(s): The most likely diagnosis in this scenario is
central retinal artery occlusion (CRAO). The sudden onset of visual loss, the
presence of an afferent pupillary defect, and red spot (the cherry red spot) on
fundoscopy are all consistent with this diagnosis. The patient also has risk
factors for this diagnosis. In branch retinal vein occlusion patients typically
have multiple retinal haemorrhages in the distribution of the vein. Macular
degeneration does not cause sudden onset visual loss and although both
retinal detachment and anterior ischaemic optic neuropathy cause acute visual
loss neither have the fundoscopic findings described.

© Medical Schools Council 2024 – reviewed August 2024


Page 38 of 104
37. A 72 year old woman has had inability to sleep well for the past 3 years. She
gets to sleep by 23:00 but wakes up two or three times in the night and gets
up by 07:00. Her husband says that she doesn't snore. She carries out her
normal daytime activities with no daytime somnolence. She is otherwise well.

Her BMI is 23 kg/m2. Her MMSE (Mini Mental State Examination) score is
27/30.

Which is the most likely cause of her insomnia?

A. Depression
B. Early stages of dementia
C. Hypomania
D. Normal age related sleep pattern
E. Obstructive sleep apnoea
Correct Answer(s): D

Justification for correct answer(s): Based on the information given, the most likely
cause of her insomnia is normal age-related sleep pattern. This is because she
is able to carry out normal daytime activities with no daytime somnolence, has
no history of snoring or other sleep-related symptoms, and has a high MMSE
score indicating good cognitive function. It is common for older adults to
experience changes in their sleep patterns, such as more fragmented sleep
and more frequent awakenings during the night. Other potential causes such
as depression, dementia, hypomania, and obstructive sleep apnoea would
require further evaluation and additional symptoms or risk factors to be
confirmed.

© Medical Schools Council 2024 – reviewed August 2024


Page 39 of 104
38. A 35 year old man attends his GP with 3 days of a red, painful left eye with no
discharge.

There is a diffuse area of redness in the medial aspect of his left sclera. His
pupils and visual acuity are normal.

Which is the most appropriate management?

A. Arrange assessment in emergency eye clinic


B. Prescribe chloramphenicol eye drops
C. Prescribe corticosteroid eye drops
D. Prescribe topical aciclovir
E. Reassure patient that it will resolve spontaneously
Correct Answer(s): A

Justification for correct answer(s): The symptoms of a painful red eye without
discharge and a diffuse area of redness on the sclera are suggestive of
scleritis, which requires immediate referral by his GP to an ophthalmologist via
the emergency eye clinic. Scleritis is an inflammatory condition of the sclera
that can lead to other serious ocular complications if untreated. Although
topical corticosteroids may be used in the management of scleritis these
should only be initiated under the supervision of an Ophthalmologist after
confirmation of the diagnosis. None of the other options in this case would be
appropriate.

© Medical Schools Council 2024 – reviewed August 2024


Page 40 of 104
39. An 18 year old woman is found dead, sitting in front of a gas fire that is still
burning. The flue that carries gases away from the fire is found to be blocked.

Which is the principal mechanism of action of the poison involved in her death?

A. Binding to the site on haemoglobin normally occupied by oxygen


B. Converting carbon dioxide to carbonic acid in the cytoplasm of peripheral cells
C. Converting the iron in haem to an iron salt
D. Damaging the lipid bilayer of alveolar pneumocytes
E. Inhibiting cytochrome enzyme systems
Correct Answer(s): A

Justification for correct answer(s): The most likely cause of death in this case is
carbon monoxide (CO) poisoning, which is known to result from blocked flues
and unvented fires. CO binds to the site on haemoglobin normally occupied by
oxygen, thereby reducing the oxygen-carrying capacity of the blood. This leads
to tissue hypoxia and eventually death.

© Medical Schools Council 2024 – reviewed August 2024


Page 41 of 104
40. A 35 year old man has burning pain in his feet and difficulty sleeping. He has
type 1 diabetes mellitus, retinopathy and nephropathy.

Investigation:eGFR 28 mL/min/1.73m2(> 60)

Which is the most appropriate management?

A. Acupuncture
B. Amitriptyline
C. Duloxetine
D. Physiotherapy
E. Sodium valproate
Correct Answer(s): B

Justification for correct answer(s): The most appropriate management for this
patient with type 1 diabetes mellitus, burning pain in his feet, difficulty
sleeping, and decreased eGFR would be amitriptyline. Although duloxetine can
be used in this condition it is not recommended with an eGFR <30 mL/min.
Clinical guideline [CG173] Published: 20 November 2013 Last updated: 22
September 2020
https://www.nice.org.uk/guidance/cg173/chapter/Recommendations

© Medical Schools Council 2024 – reviewed August 2024


Page 42 of 104
41. A 24 year old woman has tiredness, bloating and weight loss with bouts of
offensive smelling diarrhoea. Abdominal examination is normal.

Investigations:
Haemoglobin 10.0 g/L (115–150)
Mean cell volume (MCV) 78 fL (80–96)
Platelets 350 × 109/L (150–400)
Duodenal biopsy shows flattening of villi and increased lymphocytes in the
lamina propria and surface epithelium. In addition, there is gross crypt
hyperplasia.

Which is the most likely diagnosis?

A. Carcinoid tumour
B. Coeliac disease
C. Collagenous enteropathy
D. Crohn's disease
E. Pseudomembranous enteropathy
Correct Answer(s): B

Justification for correct answer(s): The most likely diagnosis is Coeliac disease.
The symptoms of tiredness, bloating and weight loss along with flattened villi
and increased lymphocytes in the lamina propria and surface epithelium on
duodenal biopsy suggest the diagnosis. The low haemoglobin and MCV values
could also be attributed to malabsorption associated with coeliac disease. The
other options listed (carcinoid tumour, collagenous enteropathy, Crohn’s
disease, and pseudomembranous enteropathy) do not fit the clinical picture
and findings described in the case.

© Medical Schools Council 2024 – reviewed August 2024


Page 43 of 104
42. An 87 year old man develops profuse watery diarrhoea 6 days after admission
for an infective exacerbation of COPD. He is currently taking oral co-
amoxiclav.

Which is the most likely causative organism?

A. Campylobacter jejuni
B. Clostridioides difficile (Clostridium difficile)
C. Escherichia coli
D. Norovirus
E. Salmonella enteriditis
Correct Answer(s): B

Justification for correct answer(s): The most likely causative organism in this case
is Clostridioides difficile (Clostridium difficile) as it is a common cause of
antibiotic-associated diarrhoea and the patient is taking oral co-amoxiclav.

© Medical Schools Council 2024 – reviewed August 2024


Page 44 of 104
43. A 22 year old woman has worsening shortness of breath and cough
productive of four to five tablespoons of sputum per day. She had childhood
pneumonia and recurrent chest infections. She coughed up blood on two
occasions many years ago.

She has bilateral scattered wheezes and coarse inspiratory crackles.

Which is the most likely diagnosis?

A. Bronchiectasis
B. COPD
C. Lung cancer
D. Pulmonary fibrosis
E. Sarcoidosis
Correct Answer(s): A

Justification for correct answer(s): Bronchiectasis is most likely due to the


copious sputum production, and the history of childhood pneumonia and
recurrent chest infections. Cystic fibrosis should be considered also as a
cause of the bronchiectasis. Bronchiectasis is characterised by permanent
dilation and thickening of bronchi resulting in chronic cough with daily sputum
production and recurrent respiratory infections. Often there are coarse
crackles on examination and there may be wheeze if there is an exacerbation.
Haemoptysis can be a feature of bronchiectasis especially when there is an
exacerbation. COPD and lung cancer are very uncommon in this age group.
Pulmonary fibrosis generally has fine crackles and copious sputum production
is less common. Sarcoidosis can cause fine crackles also, but it is unusual in
this age group and it does not usually present with recurrent chest infections
and copious sputum production is not usually a feature.

© Medical Schools Council 2024 – reviewed August 2024


Page 45 of 104
44. A 52 year old woman has had four episodes of severe, colicky epigastric pain
associated with vomiting over the past 3 months. The episodes occurred after
eating and lasted for about 1 hour before complete resolution. She has type 2
diabetes mellitus and takes metformin.

Abdominal examination is normal. Her BMI is 35 kg/m2.

Which investigation is most likely to confirm the diagnosis?

A. Helicobacter stool antigen test


B. Serum Amylase
C. Plain abdominal X-ray
D. Ultrasonography of abdomen
E. Upper gastrointestinal endoscopy
Correct Answer(s): D

Justification for correct answer(s): The patient reports intermittent episodes of


colicky pain in association with nausea and vomiting. This would be consistent
with biliary colic. She has risk factors for gallstones including being female,
middle aged and overweight. Therefore an ultrasound of abdomen would be
the appropriate investigation at this stage.
Pain associated with gastro-oesophageal reflux disease (GORD) and dyspepsia
– this is likely to lead to persistent symptoms over a period of time. Upper Gi
endoscopy and Helicobacter stool antigen test are relevant when investigating
for suspected GORD. Serum amylase is indicated in suspected pancreatitis.
Plain abdominal X-ray would be potentially helpful in the investigation of
suspected small or large bowel pathologies (e.g. obstruction, constipation),
although CT would likely be more informative.

© Medical Schools Council 2024 – reviewed August 2024


Page 46 of 104
45. An 83 year old woman has recurring dizzy spells. The episodes are
associated with transient shaking of her hands that is most noticeable before
her lunch and evening meals. She has hypertension and type 2 diabetes
mellitus. She takes metformin (1 g twice daily), gliclazide (80 mg twice daily)
and ramipril (10 mg daily).

Her BP is 138/82 mmHg lying and 130/78 mmHg standing. Her blood capillary
glucose is 6 mmol/L.

Investigations:
Sodium 136 mmol/L (135-146)
Potassium 5.0 mmol/L (3.5-5.3)
Urea 3.9 mmol/L (2.5-7.8)
Creatinine 77 μmol/L (60-120)
Glycated haemoglobin 50 mmol/mol (20-42)

Which is the most appropriate therapeutic change?

A. Increase gliclazide dose


B. Increase metformin dose
C. Reduce gliclazide dose
D. Reduce metformin dose
E. Reduce ramipril dose
Correct Answer(s): C

Justification for correct answer(s): This 83 year old woman is experiencing


symptoms consistent with hypoglycaemia. The timing of her ‘dizzy spells’ and
hand shaking - before lunch and evening meals - suggests they might be
related to periods of extended fasting and thus to her medications, particularly
the ones that have the potential to lower blood glucose. Furthermore, her
glycated haemoglobin (HbA1c) level is 50 mmol/mol. While this is elevated
compared to the normal reference range provided (20-42 mmol/mol), it’s in the
target range or even a bit stringent for many elderly patients with type 2
diabetes. Overly aggressive glycaaemic control can increase the risk of
hypoglycaemia in older adults. Therefore, the most appropriate therapeutic
change, given the symptoms and the clinical context, would be to reduce
gliclazide dose.

© Medical Schools Council 2024 – reviewed August 2024


Page 47 of 104
46. A 76 year old woman has progressive breathlessness with right-sided pleuritic
chest pain and weight loss over 6 months. She is a retired mechanic and has
a 25 pack-year smoking history.

Investigations:
Chest X-ray: marked volume loss in right hemithorax

CT scan of chest: see image

Which is the most likely diagnosis?

A. Asbestosis
B. Chronic hypersensitivity pneumonitis
C. Lung cancer
D. Malignant pleural mesothelioma
E. Tuberculosis
Correct Answer(s): D

Justification for correct answer(s): The symptoms and imaging findings described
in the scenario are consistent with malignant pleural mesothelioma. The CT
image shows marked volume loss in the right lung and the right lung is
encased with tumour. The patient may have been exposed to asbestos in her
job as a mechanic (brake pads etc). Chronic hypersensitivity pneumonitis and
asbestosis have changes in the lung fields, not the pleura. The changes are
usually bilateral and crackles are heard at the area of abnormality. This
patient’s history would fit with lung cancer and she does have risk factors for
lung cancer (smoking and asbestos exposure) but chest pain is more common
© Medical Schools Council 2024 – reviewed August 2024
Page 48 of 104
with mesothelioma and the CT scan appearances are classical of
mesothelioma as the thickening is of the pleura. TB can mimic anything but it
is less likely in this case and there is no fever.

© Medical Schools Council 2024 – reviewed August 2024


Page 49 of 104
47. A 65 year old woman has severe left-sided abdominal pain. Yesterday, she
noticed blood mixed in with her stools. She has no weight loss.

Her temperature is 37.7°C. She is very tender on palpation in the left lower
quadrant. No masses are felt on rectal examination, but there is blood on the
glove.

Which is the most likely cause of her symptoms?

A. Angiodysplasia
B. Colorectal cancer
C. Diverticulitis
D. Haemorrhoids
E. Ulcerative colitis
Correct Answer(s): C

Justification for correct answer(s): Based on the patient's presentation of severe


left-sided abdominal pain, blood mixed in with her stools, and tenderness on
palpation in the left lower quadrant, the most likely cause of her symptoms is
diverticulitis.
Angiodysplasia is a condition where abnormal blood vessels in the
gastrointestinal tract can cause bleeding, but it typically presents with painless
bleeding and is not associated with abdominal pain or tenderness.
Colorectal cancer can cause abdominal pain and bleeding, but it is less likely
to present with acute onset of severe pain and tenderness, especially if there is
no history of weight loss.
Haemorrhoids can cause bleeding during bowel movements, but they typically
do not cause severe abdominal pain.
Ulcerative colitis is a chronic inflammatory bowel disease that can cause
abdominal pain and bloody diarrhoea, but it is less likely to present with acute
onset of severe pain and tenderness. Additionally, the absence of any history
of weight loss makes ulcerative colitis less likely.
Therefore, based on the information provided, diverticulitis is the most likely
cause of this patient's symptoms.

© Medical Schools Council 2024 – reviewed August 2024


Page 50 of 104
48. A 35 year old woman was admitted two days ago after taking 32 paracetamol
tablets. She has alcohol use disorder.

Her weight is 40 kg.

She has been treated with a full dose of acetylcysteine.

Which investigation best demonstrates restoration of liver synthetic function?

A. Albumin
B. ALT
C. Bilirubin
D. γGT
E. PT
Correct Answer(s): E

Justification for correct answer(s): In this scenario, the patient has ingested a
potentially toxic amount of paracetamol and has been treated with
acetylcysteine, which is the antidote for paracetamol overdose. As a result, the
most likely organ affected is the liver, and the investigation that best
demonstrates the restoration of liver synthetic function is the prothrombin time
(PT).
Paracetamol overdose can cause liver damage, which can lead to a decrease in
the synthesis of clotting factors by the liver. The PT is a measure of the time it
takes for a clot to form in a blood sample, and it is used to assess liver
function. An elevated PT indicates impaired liver function, and a prolonged PT
is commonly seen in patients with liver damage due to paracetamol overdose.
Therefore, monitoring the PT is essential in patients with paracetamol
overdose to assess the extent of liver damage and to evaluate the
effectiveness of treatment with acetylcysteine.
Albumin is a protein synthesized by the liver and is often used as a marker of
liver function. However, albumin levels may not show immediate changes in
liver synthetic function in the setting of acute liver injury.
ALT is an enzyme that is released into the bloodstream when liver cells are
damaged. ALT levels can be elevated in patients with liver damage due to
paracetamol overdose, but they do not reflect the restoration of liver synthetic
function.
Bilirubin is a pigment produced by the breakdown of red blood cells and is
typically elevated in patients with liver damage. However, it does not reflect the
restoration of liver synthetic function.
γGT is an enzyme found in liver cells that can be elevated in patients with liver
damage. However, it is not a specific marker of liver function, and its levels
may remain elevated even after the restoration of liver synthetic function.
Therefore, the investigation that best demonstrates the restoration of liver
synthetic function in this patient is the prothrombin time (PT).

© Medical Schools Council 2024 – reviewed August 2024


Page 51 of 104
49. A 64 year old woman has thumping palpitations and says that she sometimes
feels her heart 'gives a sudden jump'. Her husband recently died due to
myocardial infarction.

Her pulse is 70 bpm and BP 136/80 mmHg. Her heart sounds are normal.

Her 12-lead ECG is shown in the tracing.

Which is the most likely cause of her palpitations?

A. Atrial fibrillation
B. Premature supraventricular beats
C. Premature ventricular beats
D. Sinus arrhythmia
E. Ventricular tachycardia
Correct Answer(s): C

Justification for correct answer(s): Premature ventricular beats (aka ventricular


ectopic beats) are a common cause of palpitations and may present with
thumping palpitations or a sensation of a sudden jump in the heart. Premature
supraventricular beats can also present in this way but the ECG shows three
broad-complex ectopic beats suggesting a ventricular origin. Premature
ventricular beats are caused by early depolarisation of the ventricular tissue
leading to an early contraction. The symptoms are usually brief and self-
limited, and the patient may not require any specific treatment. However, it is
important to exclude underlying cardiac disease.

Atrial fibrillation and ventricular tachycardia can also cause palpitations but
are less likely in this scenario based on the patient’s history, normal physical
examination findings and abnormal ECG. Sinus arrhythmia is a normal
variation in heart rate that occurs during breathing and is not typically
associated with palpitations or sudden jumps in heart rate.

© Medical Schools Council 2024 – reviewed August 2024


Page 52 of 104
50. A 35 year old man has painless swelling of the right side of his scrotum.

The swelling is soft and fluctuant, and transilluminates.

Which is the most likely diagnosis?

A. Hydrocoele
B. Inguinal hernia
C. Testicular torsion
D. Testicular tumour
E. Varicocoele
Correct Answer(s): A

Justification for correct answer(s): Based on the provided information, the most
likely diagnosis is hydrocoele. A hydrocoele is a collection of fluid that
surrounds the testicle within the tunica vaginalis, causing painless swelling of
the scrotum. The swelling is typically soft, fluctuant, and transilluminates when
a light is shone on it. This condition can occur at any age, but it is more
common in older men. An inguinal hernia may also present as a painless
swelling in the scrotum, but it is typically firmer and does not transilluminate.
Testicular torsion, on the other hand, is a painful condition that typically
presents with sudden onset of severe testicular pain, often accompanied by
nausea and vomiting. Testicular tumours may present as a painless testicular
mass or swelling, but they are less likely to cause diffuse scrotal swelling.
Varicoceles are enlarged veins within the scrotum, but they typically feel like a
bag of worms and do not transilluminate.

© Medical Schools Council 2024 – reviewed August 2024


Page 53 of 104
51. A 67 year old woman has an ulcer with a raised white margin on her left ear.
The lesion has been present for 3 years, growing slowly and never completely
healing. She spent 20 years living in Australia before returning to the UK
recently.

She has a small ulcerated area, 4 mm × 6 mm, on her left pinna.

Which is the most likely diagnosis?

A. Actinic keratosis
B. Basal cell carcinoma
C. Malignant melanoma
D. Seborrhoeic keratosis
E. Squamous cell carcinoma
Correct Answer(s): B

Justification for correct answer(s): The history of an ulcerated lesion on the ear in
an individual likely to have had a high level of ultraviolet light exposure from
living in Australia should raise the possibility of a keratinocyte cancer. Given
the long history yet small size of the lesion, together with the description of a
raised, pale border make basal cell carcinoma (BCC) the most likely diagnosis.
Other characteristic features would be a shiny or ‘pearly’ surface, a rolled edge
or overlying telangiectasia.
Actinic keratoses are pink and scaly or hyperkeratotic and do not ulcerate.
There is no pigmentation to suggest melanoma. Whilst amelanotic melanoma
is not completely impossible here, BCC is hugely more common and therefore
a much more likely diagnosis. Seborrhoeic keratosis is a harmless warty
lesion, which is usually pigmented and does not ulcerate. Squamous cell
carcinoma is the other main type of keratinocyte cancer, but is usually red, not
pale. It typically grows at a much faster rate than BCC.

© Medical Schools Council 2024 – reviewed August 2024


Page 54 of 104
52. A 90 year old man has had 3 days of spasmodic suprapubic pain radiating to
the tip of the penis. His long-term urinary catheter has recently been changed.
He is mildly confused.

His temperature is 38.2º C, pulse rate 88 bpm, BP 146/88 mmHg, respiratory


rate 15 breaths per minute and oxygen saturation 96% breathing
air. Urinalysis: dark and strong smelling, protein 1+, blood 1+, negative for
leucocytes and nitrites.

Which factor(s) indicate(s) the need to start antibiotics?

A. Blood and protein in urine


B. Dark strong-smelling urine
C. Fever and mild confusion
D. Presence of pain
E. Recent catheter change
Correct Answer(s): C

Justification for correct answer(s): The patient’s fever, mild confusion and urinary
symptoms suggest the presence of a systemic infection, which could be due to
a urinary tract infection (UTI) or catheter-related infection given his recent
catheter change. It is therefore important to start antibiotics. The presence of
blood and protein in the urine, are not specific to urinary infection and are very
commonly associated with indwelling urinary catheters. Blood and protein in
the urine could also suggest other renal or urinary tract pathologies. Dark
strong-smelling urine is again not specific for urinary infection. Pain could be a
symptom of urinary tract pathology, but it does not necessarily indicate the
need for antibiotics unless it is associated with other signs of infection. he
indications for antibiotics with a catheter change include neutrophils < 1 x
109/L, multiple attempts or traumatic insertion, post trans-urethral urological
surgery, previous episode of catheter change related sepsis, frank pus at the
urethral meatus or in critical care patients.

© Medical Schools Council 2024 – reviewed August 2024


Page 55 of 104
53. A 68 year old woman attends her GP with intermittent palpitations. She has
diet-controlled type 2 diabetes.

Her pulse rate is 78 bpm, regular. BP 121/77 mmHg.

Investigations:Haemoglobin 137 g/L (115–150)


eGFR 85 mL/min/1.73m2(> 60)

Which is the most appropriate initial management?

A. Start apixaban 5 mg twice daily


B. Start aspirin 75mg once daily
C. Start digoxin 125 micrograms once daily
D. Give digoxin 75 micrograms stat
E. Refer for left atrial ablation
Correct Answer(s): A

Justification for correct answer(s): This patient has paroxysmal atrial fibrillation.
Her CHA2DS2-VASc score is 3 and ORBIT Score is 0. Anticoagulant therapy,
such as apixaban, is recommended for patients with atrial fibrillation and a
moderate to high stroke risks. Aspirin is not recommended for stroke
prevention in atrial fibrillation, and digoxin is used primarily for rate control
rather than stroke prevention. Left atrial ablation would only be considered if
drug treatment is unsuccessful, unsuitable or not tolerated in people with
symptomatic paroxysmal or persistent atrial fibrillation.

NICE guideline [NG196] Published: 27 April 2021 Last updated: 30 June 2021
https://www.nice.org.uk/guidance/ng196/chapter/Recommendations

© Medical Schools Council 2024 – reviewed August 2024


Page 56 of 104
54. A 48 year old man has visible haematuria and right loin pain.

His temperature is 37.3°C, pulse rate 72 bpm and BP 170/97 mmHg. Masses
are palpable in both flanks.

Investigations:
Creatinine 220 µmol/L (60-120)
Urinalysis: blood 4+

Which is the most appropriate next investigation?

A. CT scan of kidneys, ureters and bladder


B. Cystoscopy
C. MR scan of renal tract
D. Ultrasound scan of renal tract
E. Urine cytology
Correct Answer(s): D

Justification for correct answer(s): The most appropriate next investigation is an


ultrasound of the renal tract. The patient likely has undiagnosed polycystic
kidney disease with bilateral renal masses, reduced renal function and
haematuria. A renal ultrasound will rapidly confirm the presence of cysts. MR
scan may be done later to assess renal sizes ahead of possible therapy with
vasopressin antagonists. Cystoscopy is not required at this stage and urine
cytology would not be of benefit. CT kidney, ureter and bladder would be used
in suspected malignancy and urolithiasis.

© Medical Schools Council 2024 – reviewed August 2024


Page 57 of 104
55. A 35 year old man attends his GP with a severe frontal headache of 12 hours'
duration. It started suddenly, reaching maximum intensity within 1 minute. He
has associated nausea. At the onset of his headache he noticed a small hole
in his vision. This hole started centrally, moved to the edge of his vision and
has now resolved. The headache is worse in bright light.

Paracetamol has not helped his pain.

Which feature should prompt immediate referral to hospital?

A. Abrupt onset
B. Failure to respond to paracetamol
C. Nausea
D. Photophobia
E. Visual disturbance
Correct Answer(s): A

Justification for correct answer(s): The case description seems to be of migraine


with a surprisingly abrupt onset of headache. It is the abrupt onset of
headache is the most worrying feature and suggests a serious underlying
cause. Abrupt onset of headache with visual disturbance could be due to
subarachnoid haemorrhage (possibly a haemorrhage into the occipital lobe,
e.g. from an intracerebral arteriovenous malformation), or haemorrhage into a
pituitary macroadenoma with compression of the anterior visual
pathway. Other possibilities include reversible cerebral vasoconstriction
syndrome, cerebral venous sinus thrombosis or low-pressure headache,
though visual disturbance would not be easily explained).

© Medical Schools Council 2024 – reviewed August 2024


Page 58 of 104
56. A 28 year old man has an insurance medical.

His pulse is 72 bpm and BP 210/110 mmHg. There is radiofemoral delay. A


systolic murmur is audible on auscultation.

Chest X-ray reveals notching of the ribs in the mid-clavicular line.

Which is the most likely diagnosis?

A. Coarctation of the aorta


B. Dissecting aortic aneurysm
C. Marfan's syndrome
D. Renal artery stenosis
E. Takayasu's arteritis
Correct Answer(s): A

Justification for correct answer(s): The patient's elevated blood pressure,


radiofemoral delay, and chest X-ray findings of notching of the ribs in the mid-
clavicular line are suggestive of aortic coarctation.
Coarctation of the aorta is a congenital defect that results in a narrowing of the
aorta, just beyond the origin of left subclavian artery. This can lead to
hypertension and other cardiovascular complications. The systolic murmur
heard on auscultation may arise from flow across the coarctation itself or
associated aortic valve disease.
Dissecting aortic aneurysm is less likely, as it typically presents with sudden,
severe chest or back pain and is a medical emergency requiring immediate
intervention. Marfan's syndrome is a genetic disorder that can affect multiple
organ systems, including the heart and blood vessels, but typically presents
with other clinical features such as joint hypermobility and tall stature. Renal
artery stenosis and Takayasu's arteritis may also cause hypertension but are
less likely given the patient's other clinical findings.

© Medical Schools Council 2024 – reviewed August 2024


Page 59 of 104
57. A 42 year old woman has two episodes of haemoptysis. She also reports two
months of sinusitis with pain, stuffiness and nose bleeds. She has some
loosening of her teeth and painful oral ulceration. Previously, she noticed a
rash and nodules over her elbows but they have disappeared.

Investigations:

Chest X-ray: see image


Urinalysis: occasional red cell casts

Which is the most likely diagnosis?

A. Behçet's disease
B. Granulomatosis with polyangiitis
C. Metastatic nasopharyngeal carcinoma
D. Syphilis
E. Tuberculosis
Correct Answer(s): B

Justification for correct answer(s): Granulomatosis with polyangiitis (GPA) is the


most likely diagnosis based on the history of sinusitis, oral ulceration and
haemoptysis and the bilateral pulmonary nodules on chest X-ray. GPA is a rare
autoimmune disease that affects small blood vessels in the body, causing
inflammation and tissue damage. The presence of red cell casts in the

© Medical Schools Council 2024 – reviewed August 2024


Page 60 of 104
urinalysis suggests renal involvement, which is also common in GPA. The
other conditions listed are less likely to explain the patient’s symptoms and
findings.

© Medical Schools Council 2024 – reviewed August 2024


Page 61 of 104
58. A 19 year old man has had a sore throat, malaise and intermittent fever for 2
weeks.

His pulse rate is 74 bpm and BP 115/75 mmHg. His throat is red and tonsils
are swollen. His sclerae are yellow-tinged. There are multiple soft palpable
lymph nodes in the neck. There is tenderness in the right upper abdominal
quadrant.

Which is the most appropriate diagnostic investigation?

A. Epstein–Barr virus test


B. HIV test
C. Lymph node biopsy for histology
D. Mantoux test
E. Throat swab and culture
Correct Answer(s): A

Justification for correct answer(s): Based on the presented information, the most
appropriate diagnostic investigation is an Epstein-Barr virus (EBV) test. The
clinical features, including sore throat, malaise, intermittent fever, swollen
tonsils, yellow-tinged sclerae, and tender lymph nodes, suggest infectious
mononucleosis (glandular fever). This is a relatively common disease in the 15
- 25 year old age group and is caused by EBV. Blood tests for antibodies can
be used to confirm acute infection once a person has been ill for at least 7
days.

© Medical Schools Council 2024 – reviewed August 2024


Page 62 of 104
59. A 27 year old woman has had abdominal pain for 48 hours. She also reports
recurrent mouth ulcers and altered bowel habit for a few weeks.

Her temperature is 37.5°C. She has central abdominal tenderness.

Investigations:
CT colonoscopy shows a normal appendix with distal small bowel thickening.
There are enlarged nodes in the small bowel mesentery.

Which is the most likely diagnosis?

A. Crohn's ileitis
B. Intestinal tuberculosis
C. Meckel's diverticulitis
D. Mesenteric adenitis
E. Small bowel lymphoma
Correct Answer(s): A

Justification for correct answer(s): Crohn’s ileitis is the most likely diagnosis
based on the patient’s symptoms, findings on CT colonoscopy, and
demographic factors. Crohn’s disease is a chronic inflammatory bowel disease
that can affect any part of the gastrointestinal tract, but it most commonly
involves the terminal ileum. The patient’s symptoms of recurrent mouth ulcers
and altered bowel habit are consistent with Crohn’s disease, and the finding of
small bowel thickening with enlarged mesenteric nodes on CT colonoscopy is
also suggestive. Intestinal tuberculosis may be considered in the differential
diagnosis, but the patient’s demographics make this less likely. Meckel’s
diverticulitis and mesenteric adenitis may also be considered, but the lack of a
diverticulum or focal lymphadenopathy makes these less likely. Small bowel
lymphoma is another possible diagnosis, but the presence of a normal
appendix makes this less likely.

© Medical Schools Council 2024 – reviewed August 2024


Page 63 of 104
60. A 34 year old man has cough and weight loss.

A diagnosis of tuberculosis is confirmed and treatment is started. As part of


his monitoring, he is screened for loss of visual acuity.

Which antituberculosis drug is an indication for visual monitoring?

A. Ethambutol hydrochloride
B. Isoniazid
C. Moxifloxacin
D. Pyrazinamide
E. Rifampicin
Correct Answer(s): A

Justification for correct answer(s): Ethambutol hydrochloride is the


antituberculosis drug that requires monitoring of visual acuity. Ethambutol can
cause optic neuritis, which can lead to impaired colour discrimination, central
visual field defects, and blurred vision. Therefore, it is recommended to
perform baseline ophthalmologic examination before starting the treatment
and periodic monitoring during the treatment, especially in patients with pre-
existing visual impairment or renal impairment. The other antituberculosis
drugs listed do not require routine ophthalmologic monitoring.

© Medical Schools Council 2024 – reviewed August 2024


Page 64 of 104
61. A 21 year old woman attends her GP with moderately severe acne. She has
tried topical retinoids and topical antibiotics without satisfactory response. She
previously had a deep vein thrombosis following a long-haul flight.

Which is the most appropriate treatment?

A. Co-cyprindiol
B. Desogestrel
C. Flucloxacillin
D. Isotretinoin
E. Lymecycline
Correct Answer(s): E

Justification for correct answer(s): The most appropriate treatment next treatment
for moderately severe acne in a patient with a history of deep vein thrombosis
is Lymecycline. Co-cyprindiol is a form of the oral contraceptive pill (OCP) with
anti-androgenic effects, which is licensed for acne. However, it carries a higher
risk of thromboembolic disease than other OCPs and is contraindicated here.
Desogestrel (when used alone) is a progesterone-only OCP, which may
exacerbate acne. Flucloxacillin has no effect on acne. Oral isotretinoin is the
most effective treatment for acne but on account of its potential adverse
effects is usually reserved for severe or scarring disease or when other
treatments including tetracycline antibiotics - have not been sufficiently
effective.

NICE guideline [NG198] Published: 25 June 2021 Last updated: 07 December


2023
https://www.nice.org.uk/guidance/ng198/chapter/RecommendationsNICE
guideline [NG198] Published: 25 June 2021 Last updated: 07 December 2023
https://www.nice.org.uk/guidance/ng198/chapter/Recommendations

© Medical Schools Council 2024 – reviewed August 2024


Page 65 of 104
62. A 45 year old man has had weight loss, fatigue and polyuria for 3 months. He
takes a number of multivitamin preparations.

Clinical examination is unremarkable.

Investigations:
Serum corrected calcium 2.9 mmol/L (2.2–2.6)Phosphate 0.82 mmol/L (0.8–
1.5)
Serum alkaline phosphatase 154 IU/L (25–115)
Parathyroid hormone 7.9 pmol/L (1.6–8.5)

Serum electrolytes and urea are normal.

Which is the most likely diagnosis?

A. Bony metastases
B. Excess calcium intake
C. Primary hyperparathyroidism
D. Sarcoidosis
E. Vitamin D excess
Correct Answer(s): C

Justification for correct answer(s): The most likely diagnosis in this case is
primary hyperparathyroidism as it is characterised by increased serum calcium
and alkaline phosphatase. The parathyroid hormone is only slightly elevated
which is still consistent with the diagnosis, as it should be suppressed in the
presence of hypercalcaemia. Excess calcium intake and vitamin D excess can
also cause hypercalcaemia but they are less likely in this case as the patient
does not report any excessive intake of these substances. Bony metastases
and sarcoidosis can also cause hypercalcemia, but they would cause a
suppressed PTH.

© Medical Schools Council 2024 – reviewed August 2024


Page 66 of 104
63. A 59 year old man has a 1 year of erectile dysfunction. He has angina, type 2
diabetes mellitus and peripheral vascular disease. He had a thyroidectomy 2
years ago for thyrotoxicosis. He takes aspirin, diltiazem, levothyroxine,
metformin, ramipril and simvastatin.

His BP is 140/90 mmHg lying and 135/85 mmHg standing. His foot pulses are
not palpable. He has normal sensation in his feet.

Which is most likely to be the main cause of his erectile dysfunction?

A. Adverse effect of medication


B. Autonomic neuropathy
C. Hypothyroidism
D. Testosterone deficiency
E. Vascular insufficiency
Correct Answer(s): E

Justification for correct answer(s): Vascular insufficiency is the most likely main
cause of erectile dysfunction in this patient. The patient has peripheral
vascular disease and similar vascular disease can occur in the penile blood
supply. None of his medications is likely to cause erectile dysfunction and
there are no clinical features of testosterone deficiency. Autonomic neuropathy
can contribute to erectile dysfunction in patients with diabetes but the patient
does not have any other features of neuropathic disease. Hypothyroidism can
also cause erectile dysfunction, but this is less likely in this patient given that
he is on levothyroxine replacement therapy.

© Medical Schools Council 2024 – reviewed August 2024


Page 67 of 104
64. A 52 year old man has had 3 months of fatigue. He has ulcerative colitis and
takes mesalazine. He drinks 20 units of alcohol per week.

His temperature is 36.8°C and pulse rate 80 bpm. He has 3 cm


hepatomegaly.

Investigations:
Albumin 36 g/L (35–50)
ALT 65 IU/L (10–50)
ALP 580 IU/L (25–115)
Bilirubin 18 µmol/L (<17)
γGT 230 IU/L (9–40)

Which is the most likely diagnosis?

A. Alcoholic hepatitis
B. Cholangiocarcinoma
C. Choledocholithiasis
D. Hepatocellular carcinoma
E. Primary sclerosing cholangitis
Correct Answer(s): E

Justification for correct answer(s): The correct answer is Primary Sclerosing


cholangitis. He has had non-specific symptoms for a number of months. His
liver function test show a cholestatic pattern with a raised ALP. This is an
increased incidence of primary sclerosing cholangitis in patients with
inflammatory bowel disease.

While alcoholic hepatitis is a possibility an alcohol intake of 20 units per week


is not high enough to cause significant damage in an otherwise healthy liver.

Cholangiocarcinoma and hepatocellular carcinoma are less likely given the


short duration of symptoms and lack of specific risk factors, such as viral
hepatitis or cirrhosis. Choledocholithiasis may present with elevated liver
function tests, but is less likely to cause hepatomegaly or fatigue.

© Medical Schools Council 2024 – reviewed August 2024


Page 68 of 104
65. A 73 year old man has increasing breathlessness over 1 week. He has
chronic kidney disease and ischaemic heart disease. He takes alfacalcidol,
aspirin, atorvastatin, bisoprolol fumarate, furosemide and irbesartan.

There are bibasal inspiratory crepitations and mild peripheral oedema.

Investigations:
Sodium 134 mmol/L (135–146)
Potassium 6.7 mmol/L (3.5–5.3)
Urea19 mmol/L (2.5–7.8)
Creatinine 259 µmol/L (60–120)
eGFR 23 mL/min/1.73 m2 (>60)

Which drug is most likely to be contributing to his hyperkalaemia?

A. Alfacalcidol
B. Aspirin
C. Bisoprolol fumarate
D. Furosemide
E. Irbesartan
Correct Answer(s): E

Justification for correct answer(s): The most likely drug contributing to the
patient's hyperkalaemia is irbesartan. Irbesartan is an angiotensin II receptor
blocker (ARB) commonly used to treat hypertension, especially in CKD. It leads
to reduced aldosterone secretion, which in turn leads to reduced potassium
secretion.

© Medical Schools Council 2024 – reviewed August 2024


Page 69 of 104
66. A 65 year old man is invited to the abdominal aortic aneurysm screening
programme.

An ultrasound scan shows his abdominal aorta to be 33 mm in diameter.

Which is the most appropriate management plan?

A. Reassure and discharge


B. Refer for angiography
C. Refer for vascular surgery
D. Repeat ultrasound scan in 12 months
E. Request CT scan of abdomen
Correct Answer(s): D

Justification for correct answer(s): Abdominal aortic aneurysm (AAA) is defined as


a permanent dilatation of the abdominal aorta with a diameter of 3 cm or more.
The risk of AAA rupture increases with increasing diameter. In the UK,
screening for AAA is offered to all men aged 65 years and above. The aorta
should be measured at three levels: just below the origin of the renal arteries,
at the level of the superior mesenteric artery, and just above the bifurcation.

Current NICE guidance recommends that if the abdominal aorta measures


between 3.0 and 4.4 cm in diameter, then an ultrasound scan should be
repeated every year. If the aorta measures between 4.5 and 5.4 cm in diameter,
then an ultrasound scan should be repeated every three months. If the aorta
measures 5.5 cm or more in diameter, then the patient should be referred for
vascular surgery. In this case, as the diameter of the aorta is 33 mm, which is
between 3.0 and 4.4 cm, the most appropriate management plan is to repeat
the ultrasound scan in 12 months.

NHS abdominal aortic aneurysm (AAA) screening programme: care pathway

Updated 16 June 2021


https://www.gov.uk/government/publications/abdominal-aortic-aneurysm-
screening-care-pathway/nhs-abdominal-aortic-aneurysm-aaa-screening-
programme-care-pathway

© Medical Schools Council 2024 – reviewed August 2024


Page 70 of 104
67. A 28 year old woman has pain on swallowing. She has asthma that is well
controlled using metered dose salbutamol and beclometasone dipropionate
(800 micrograms/day) inhalers.

She has white plaques in her mouth.

An anti-fungal oral suspension is prescribed.

What is the most appropriate management with regard to her beclometasone?

A. Change beclometasone dipropionate to a dry powder formulation


B. Change beclometasone dipropionate to fluticasone
C. Change beclometasone dipriopionate to salmeterol
D. Take beclometasone dipropionate using a large volume spacer
E. Take salbutamol and beclometasone dipropionate at least 1h apart
Correct Answer(s): D

Justification for correct answer(s): This patient has developed oral candidiasis
and this is most likely due to local deposition of the inhaled steroid
(beclometasone dipropionate). The risk of this happening again can be
reduced by using a large volume spacer as there will be less local deposition
of the drug in her mouth. Changing to a dry powder or a different steroid
inhaler is unlikely to help and may make things worse.

© Medical Schools Council 2024 – reviewed August 2024


Page 71 of 104
68. A 37 year old man has increasing fever and shortness of breath. He was
admitted 3 days ago with left lower lobe pneumonia due penicillin-sensitive
Streptococcus pneumoniae. He is receiving intravenous benzylpenicillin 1.2 g
four times daily.

His temperature is 38.5°C, pulse 100 bpm and BP 122/80 mmHg.

Investigations:
Chest X-ray: left basal effusion.

Diagnostic pleural aspiration: Purulent fluid. Microscopy shows numerous


polymorphs and Gram-positive cocci

Which is the most appropriate next step in management?

A. Chest drain insertion


B. Increase dose of benzylpenicillin
C. Open thoracostomy
D. Switch benzylpenicillin to ceftriaxone
E. Switch benzylpenicillin to vancomycin
Correct Answer(s): A

Justification for correct answer(s): This patient has a fever and an empyema at the
left lung base. A chest drain needs to be inserted to allow the purulent fluid to
be drained away. The patient is already on appropriate antibiotics for
Streptococcus pneumoniae but a discussion with the Microbiology team would
still be useful.

© Medical Schools Council 2024 – reviewed August 2024


Page 72 of 104
69. A 52 year old man has 4 weeks of joint pain, fever and weight loss. He is a
non-smoker and has no significant medical history.

Examination is unremarkable.

Investigations:
Calcium 3.12 mmol/L (2.2–2.6)
Plasma parathyroid hormone <0.5 pmol/L (0.9–5.4)

Chest X-ray shows perihilar lymphadenopathy.

Which is the most likely diagnosis?

A. Granulomatosis with polyangiitis


B. Hodgkin's lymphoma
C. Primary hyperparathyroidism
D. Sarcoidosis
E. Tuberculosis
Correct Answer(s): D

Justification for correct answer(s): The most likely diagnosis is sarcoidosis due to
the history, elevated calcium and perihilar lymphadenopathy. The elevated
calcium and low parathyroid hormone can occur in sarcoidosis due to
increased production of 1,25-dihydroxyvitamin D by activated macrophages in
the granulomas. Hodgkin’s lymphoma and tuberculosis can also cause
lymphadenopathy, weight loss and fever but hypercalcaemia is less likely. The
normal plasma parathyroid hormone level makes primary hyperparathyroidism
less likely. Granulomatosis with polyangiitis may present with joint pain and
fever but does not usually cause hypercalcaemia.

© Medical Schools Council 2024 – reviewed August 2024


Page 73 of 104
70. A 56 year old man has had a single episode of painless visible haematuria.
He has no other urinary symptoms and is otherwise fit and well. He has
smoked ten cigarettes per day for the past 35 years.

He has a BP of 140/85 mmHg. Urinalysis performed after this episode shows


blood 2+, no protein and no nitrites.

Investigations:
Urea 6.5 mmol/L (2.5–7.8)
Creatinine 95 µmol/L (60–120)

Urine culture: no growth

Which investigation is most likely to confirm the diagnosis?

A. Flexible cystoscopy
B. Serum prostate specific antigen
C. Transrectal ultrasound scan of prostate
D. Ultrasound scan of kidneys
E. Urine cytology
Correct Answer(s): A

Justification for correct answer(s): A single episode of painless visible haematuria


in a smoker is concerning for bladder cancer. Cystoscopy will enable
visualisation and biopsy of any lesion in the bladder wall.

© Medical Schools Council 2024 – reviewed August 2024


Page 74 of 104
71. The daughter of a 78 year old man is concerned about an area of redness on
her father's back. He lives alone and spends most of his day sitting in a chair.

There is a reddened area over the sacrum, but his skin is intact.

He is considered to be at risk of pressure ulcers.

Which member of the community multidisciplinary team would be most suited to


conduct an initial assessment?

A. Dietician
B. District nurse
C. Occupational therapist
D. Physiotherapist
E. Tissue viability nurse
Correct Answer(s): B

Justification for correct answer(s): The district nurse is the most appropriate
member of the community multidisciplinary team to conduct an initial
assessment of redness over the sacrum in an elderly man who is at risk of
pressure ulcers. The district nurse will advise on care planning and give
repositioning advice. Tissue viability nurses offer support to district nurses in
the management of complex wounds. Dieticians have an important role in the
optimisation of patients at risk of pressure damage but would not carry out the
initial assessment of an area at risk of further pressure damage. Occupational
therapists’ s provide assessment and treatment to those who are finding it
difficult to carry out everyday tasks. Physiotherapists support and optimise
movement and function in patients.

© Medical Schools Council 2024 – reviewed August 2024


Page 75 of 104
72. A doctor in training sustains a needle-stick injury while inserting a venous
cannula into a patient. The patient has a history of intravenous drug use.

Which is the most important first action for the doctor in training?

A. Encourage bleeding from the needle-stick injury wound


B. Make an appointment with occupational health
C. Seek post-exposure prophylaxis
D. Test the patient for HIV
E. Get an immediate blood sample taken from themselves
Correct Answer(s): A

Justification for correct answer(s): The most important first action is to manage
the local skin puncture wound sustained by the doctor in training, so the best
answer is to encourage bleeding from the wound. Standard management of the
needlestick puncture wound states "puncture wounds should be encouraged
to bleed freely, but should not be sucked. Small wounds and punctures may
also be cleansed with an antiseptic, for example an alcohol-based hand
hygiene solution."

Post-exposure prophylaxis for HIV Last revised in May 2024

https://cks.nice.org.uk/topics/hiv-infection-aids/management/post-exposure-
prophylaxis/

© Medical Schools Council 2024 – reviewed August 2024


Page 76 of 104
73. A 32 year old woman has 3 weeks of fever, rigors and lethargy. In the past
week, she has also become breathless on exertion. She is an intravenous
drug user.

Her temperature is 38°C, pulse rate 100 bpm regular, and BP 100/60 mmHg.
Her JVP is raised with predominant V waves. There is a pansystolic murmur
at the left sternal edge on inspiration. She has reduced air entry with dullness
to percussion at the right lung base. She has swelling of both ankles.

Which is the most likely pathogen?

A. Enterococcus faecalis
B. Staphylococcus aureus
C. Staphylococcus epidermidis
D. Streptococcus bovis / streptococcus equinus complex
E. Streptococcus viridans
Correct Answer(s): B

Justification for correct answer(s): The patient's presentation with fever, rigors,
lethargy and breathlessness suggests sepsis. The pansystolic murmur at the
left sternal edge heard on inspiration suggests tricuspid regurgitation and thus
the most likely diagnosis is tricuspid valve endocarditis. Tricuspid
regurgitation is a common complication of right-sided infective endocarditis in
intravenous drug users. Staphylococcus aureus is a common pathogen in
intravenous drug users and can cause endocarditis, pneumonia, and sepsis.
The other organisms listed can all cause endocarditis but are less commonly
associated with intravenous drug use-related infections.

© Medical Schools Council 2024 – reviewed August 2024


Page 77 of 104
74. A 34 year old woman has sudden onset of right arm weakness and inability to
speak. She has migraines and generalised joint pains. Four years ago, she
had a deep vein thrombosis in her right leg.

Her pulse rate is 68 bpm and BP 178/94 mmHg. She has an expressive
dysphasia. She has flaccid weakness of her right arm and facial droop on the
right lower half of her face.

Investigations:
Haemoglobin 118 g/L (115–150)
White cell count 4.3 × 109/L (3.8–10.0)
Neutrophils 2.1 × 109/L (2.0–7.5)
Lymphocytes 0.6 × 109/L (1.1–3.3)
Platelets 132 × 109/L (150–400)
Total cholesterol 4.6 mmol/L (<5.0)

CT scan of head left frontoparietal infarct

Which additional investigation is most likely to reveal the underlying cause of her
stroke?

A. Anti-dsDNA antibody
B. Anticardiolipin antibody
C. Anti-Ro antibody
D. Rheumatoid factor
E. Serum immunoglobulins
Correct Answer(s): B

Justification for correct answer(s): The most likely underlying cause of her stroke
is a cardioembolic source, possibly related to her history of deep vein
thrombosis. Therefore, the most appropriate investigation to reveal the
underlying cause of her stroke is anticardiolipin antibody.
Anticardiolipin antibodies are a type of antiphospholipid antibody that can
cause thrombosis and are associated with an increased risk of stroke. Patients
with a history of deep vein thrombosis, like this patient, are at increased risk
for the development of anticardiolipin antibodies. Testing for the presence of
anticardiolipin antibodies can help confirm the diagnosis of antiphospholipid
syndrome, which is an important cause of thrombotic events, including stroke.
While the patient's history of migraine and joint pains raise the possibility of an
underlying autoimmune disorder, such as systemic lupus erythematosus, the
presence of anticardiolipin antibodies is a more specific and relevant
investigation in the context of her recent stroke.

© Medical Schools Council 2024 – reviewed August 2024


Page 78 of 104
75. A 55 year old woman has been feeling tired and sleepy. Her partner says that
she snores heavily. She has type 2 diabetes mellitus and takes metformin.

Her BMI is 38 kg/m2. Her oxygen saturation is 95% breathing air. Her Epworth
sleepiness score is 19 (normal <11). Her HbA1cis 60 mmol/mol (20-42).

Which treatment is most likely to improve her daytime somnolence?

A. Bariatric surgery
B. Continuous positive airway pressure ventilation
C. Long acting insulin
D. Mandibular advancement device
E. Modafinil
Correct Answer(s): B

Justification for correct answer(s): Based on the patient’s clinical presentation


and history, the most likely cause of the daytime somnolence is obstructive
sleep apnoea (OSA), which is characterised by snoring, excessive daytime
sleepiness, and obesity. Continuous positive airway pressure (CPAP)
ventilation is the gold standard treatment for OSA and involves using a mask
to deliver air pressure to keep the airway open during sleep. It is highly
effective in reducing daytime sleepiness, improving quality of life, and
reducing the risk of cardiovascular complications associated with untreated
OSA. While bariatric surgery may be considered in obese patients with OSA, it
is not the first-line treatment for daytime somnolence. Long-acting insulin is
also not indicated in this patient. Mandibular advancement devices may be
considered in patients with mild to moderate OSA who cannot tolerate CPAP,
but they are generally less effective than CPAP. Modafinil is a wake-promoting
agent may be used as an adjunctive therapy in patients with residual daytime
sleepiness despite optimal CPAP therapy, but it is not a first-line treatment for
OSA.

© Medical Schools Council 2024 – reviewed August 2024


Page 79 of 104
76. A 72 year old man has had six months of increased frequency of defaecation
and three months of dark red rectal bleeding mixed with the stool.

Investigations:
Haemoglobin 101 g/L (130–175)
Mean cell haemoglobin (MCH) 24 pg (27–33)
MCV 73 fL (80–96)
White cell count 9.1 x 109/L (3.0–10.0)
Platelets 354 x 109/L (150–400)

Which is the most likely diagnosis?

A. Colonic carcinoma
B. Diverticular disease
C. Haemorrhoids
D. Ischaemic colitis
E. Ulcerative colitis
Correct Answer(s): A

Justification for correct answer(s): Colonic carcinoma is the most likely diagnosis
in this patient with a six-month history of increased frequency of defecation
and three months of dark red rectal bleeding mixed with the stool. The low
haemoglobin, low MCV, and low MCH suggest that the patient has iron
deficiency anaemia, which is commonly associated with colorectal cancer. The
other differential diagnoses, such as diverticular disease, haemorrhoids,
ischaemic colitis, and ulcerative colitis, may also cause rectal bleeding, but are
less likely to present with such a prolonged duration of symptoms and iron
deficiency anaemia. A colonoscopy would be required for confirmation of the
diagnosis.

© Medical Schools Council 2024 – reviewed August 2024


Page 80 of 104
77. A 24 year old man is reviewed following a recent orchidectomy.

The pathologist's report describes a mass with cystic spaces. Histological


examination shows areas of mature cartilage and columnar epithelium.

Which is the most likely diagnosis?

A. Chondrosarcoma
B. Hamartoma
C. Lymphoma
D. Seminoma
E. Teratoma
Correct Answer(s): E

Justification for correct answer(s): The most likely diagnosis in this case is
teratoma, as it is a type of germ cell tumour that often contains different types
of tissue, including cartilage and epithelium.

© Medical Schools Council 2024 – reviewed August 2024


Page 81 of 104
78. A 50 year old man has had increasing breathlessness when climbing the
stairs. He has no chest pain, wheeze or cough. He has COPD. He takes a
salbutamol inhaler several times per day. He is an ex-smoker of 3 months and
has a 30 pack-year smoking history. His weight is stable and he is otherwise
well.

Investigation:
FEV1 : 75% predicted

Which is the most appropriate next step in pharmacological management?

A. Inhaled beclometasone
B. Inhaled tiotropium and salmeterol
C. Nebulised salbultamol and ipratropium bromide
D. Oral prednisolone
E. Oral theophylline
Correct Answer(s): B

Justification for correct answer(s): The most appropriate next step in


pharmacological management would be inhaled tiotropium and salmeterol.
This combination therapy is recommended for patients with COPD who have
persistent symptoms despite bronchodilator therapy (such as salbutamol) and
who have an FEV1 of less than 80% predicted. Tiotropium is a long-acting
anticholinergic bronchodilator, while salmeterol is a long-acting beta-agonist
bronchodilator. Together, they work to improve lung function and reduce
breathlessness in patients with COPD.

Inhaled beclometasone is a corticosteroid inhaler and oral prednisolone is a


systemic corticosteroid used for acute exacerbations of COPD rather than
chronic management. Nebulised salbutamol and ipratropium bromide is a
combination bronchodilator therapy used for acute exacerbations of COPD.
Oral theophylline is a bronchodilator that is not typically recommended as a
first-line therapy for COPD management.

NICE guideline [NG115] Published: 05 December 2018 Last updated: 26 July


2019
https://www.nice.org.uk/guidance/ng115/chapter/Recommendations

© Medical Schools Council 2024 – reviewed August 2024


Page 82 of 104
79. An 86 year old woman has had three falls in the past 3 months. On each
occasion, she describes feeling lightheaded and dizzy prior to falling. She is
taking alendronic acid, amlodipine, atorvastatin, metformin and zolpidem
tartrate.

Her BP is 132/80 mmHg sitting and 138/84 mmHg standing.

Which medication is most likely to be contributing to her falls?

A. Alendronic acid
B. Amlodipine
C. Atorvastatin
D. Metformin
E. Zolpidem tartrate
Correct Answer(s): E

Justification for correct answer(s): Alendronic acid is a bisphosphonate and is not


directly associated with falls. Atorvastatin is an HMG-CoA Reductase inhibitor
which is not directly related to falls. Metformin is a biguanide and is not
directly associated with falls. Whilst amlodipine can be associated with
postural hypotension and subsequent falls this is not the case in this patient
who does not have a postural drop in blood pressure on standing. Zolpidem
tartrate is the correct answer as it is a non-benzodiazepine hypnotic and
sedative and is associated with postural instability and falls.

© Medical Schools Council 2024 – reviewed August 2024


Page 83 of 104
80. A 81 year old man has three months of malaise, bleeding gums and pain in
his legs. He has been struggling to look after himself since his wife died one
year ago. He has a poor diet and reports that he mostly has tea and toast.

He has gingival hypertrophy and skin petechiae.

Which micronutrient deficiency is he most likely to have?

A. Magnesium
B. Vitamin A
C. Vitamin B 1
D. Vitamin C
E. Zinc
Correct Answer(s): D

Justification for correct answer(s): This is typical of vitamin C deficiency which


presents with a petechial rash and gum changes. It is still seen in UK in people
with poor diets.

© Medical Schools Council 2024 – reviewed August 2024


Page 84 of 104
81. A 45 year old man has had 6 months of tiredness, reduced libido and erectile
dysfunction.

Investigations:
Testosterone 1.8 nmol/L (9.9–27.8)
LH 1.2 U/L (1–8)
FSH 1.0 U/L (1–12)

Which is the most likely cause of his presentation?

A. Anabolic steroid misuse


B. Androgen insensitivity syndrome
C. Congenital adrenal hyperplasia
D. Klinefelter's syndrome
E. Pituitary adenoma
Correct Answer(s): E

Justification for correct answer(s): Based on the low testosterone and low LH and
FSH levels, the most likely cause of his presentation is a pituitary adenoma
leading to hypogonadotropic hypogonadism. The pituitary adenoma would
suppress the production of LH and FSH, which are required for testosterone
production in the testes.

© Medical Schools Council 2024 – reviewed August 2024


Page 85 of 104
82. A 24 year old man is admitted to hospital with an exacerbation of asthma.

His symptoms improve with treatment, and he is ready for discharge after 24
hours. His discharge medication includes a salbutamol inhaler, a combined
beclometasone and salmeterol inhaler, and a short course of oral
prednisolone.

Which further management must be provided prior to discharge?

A. Antibiotic rescue pack


B. Course of antihistamines
C. Nebuliser machine for use at home
D. Personalised asthma action plan
E. Volumatic spacer device
Correct Answer(s): D

Justification for correct answer(s): A personalised asthma action plan should be


provided prior to discharge. It is a written plan that outlines the patient's daily
management of asthma and how to adjust treatment in response to worsening
symptoms or changes in lung function. It is an essential tool for patients with
asthma to ensure that they can manage their condition effectively and prevent
future exacerbations. Antibiotic rescue packs and antihistamines are not
routinely prescribed for asthma management. Nebuliser machines and
Volumatic spacer devices may be prescribed as needed, but a personalised
asthma action plan is a more crucial component of long-term asthma
management.
NICE Guidelines Asthma Quality standard [QS25] Last updated: 20 September
2018.

© Medical Schools Council 2024 – reviewed August 2024


Page 86 of 104
83. A 40 year old man develops sudden breathlessness 5 days after an acute
inferior ST-elevation myocardial infarction treated by primary coronary
intervention.

His pulse is 110 bpm, BP 110/75 mmHg, respiratory rate 22 breaths per
minute and oxygen saturation 92% breathing 28% oxygen via Venturi mask.
There is a pansystolic murmur at the apex and bibasal inspiratory crackles.

Which is the most likely cause of this presentation?

A. Acute pulmonary embolus


B. Aortic regurgitation
C. Cardiac tamponade
D. Papillary muscle rupture
E. Pericarditis
Correct Answer(s): D

Justification for correct answer(s): Based on the clinical features described, the
most likely cause of the presentation is acute heart failure due to papillary
muscle rupture as a complication of his recent myocardial infarction. The
presence of a pansystolic murmur at the apex suggests mitral regurgitation,
which can occur due to the rupture of one of the papillary muscles that
anchors the valve leaflets. The bibasal inspiratory crackles suggest pulmonary
oedema, which can occur as a result of the increased pressure in the left
atrium and pulmonary veins due to the mitral regurgitation. While acute
pulmonary embolus, aortic regurgitation, cardiac tamponade, and pericarditis
can also cause acute breathlessness, they are less likely in this clinical
scenario.

© Medical Schools Council 2024 – reviewed August 2024


Page 87 of 104
84. A 48 year old man attends the GP surgery with headaches. He has noticed
that his hands have become larger, and his facial features have coarsened.
More recently, his vision has deteriorated.

He has an upper temporal defect in both visual fields.

Damage to which structure is the most likely source of his visual problems?

A. Lateral geniculate body


B. Occipital cortex
C. Oculomotor nerve
D. Optic chiasm
E. Optic radiation
Correct Answer(s): D

Justification for correct answer(s): This 48-year-old man’s presentation suggests


acromegaly, which is typically caused by a growth hormone-secreting pituitary
adenoma. One of the complications of a pituitary tumour is compression of
nearby structures. Given the bitemporal (upper temporal) visual field defect
described, this indicates compression of the optic chiasm. When the optic
chiasm is compressed, particularly from an inferior approach as with a
pituitary tumour, the crossing fibres (those responsible for the
peripheral/temporal vision) are primarily affected, leading to a bitemporal field
defect.

© Medical Schools Council 2024 – reviewed August 2024


Page 88 of 104
85. A 42 year old man has a rash on his face, mainly around his chin. The rash
started 24 hours ago with a 0.5 cm thin-walled blister that then ruptured,
leaving a yellow crusted lesion that has since enlarged and now other similar
lesions are appearing in the same area. He is a primary school teacher.

Which is the most likely causative organism?

A. Escherichia coli
B. Pseudomonas aeruginosa
C. Staphylococcus aureus
D. Streptococcus pyogenes
E. Varicella zoster virus
Correct Answer(s): C

Justification for correct answer(s): The description of the eruption fits best with
bullous impetigo, although this usually occurs in children. Staphylococcus
aureus, is the most common causative organism, although Streptococcus
pyogenes can also be responsible for non-bullous impetigo. E. coli is not
expected to cause skin infection. Pseudomonas may be found as a coloniser in
chronic wounds but does not cause primary cutaneous infection in
immunocompetent individuals. Varicella zoster virus causes chicken pox and
subsequently shingles, neither of which fit the clinical picture described here.

© Medical Schools Council 2024 – reviewed August 2024


Page 89 of 104
86. A 48 year old woman has had 3 years of increasing knee pain and reduced
physical activity. She has radiologically-confirmed osteoarthritis. She has
hypertension and type 2 diabetes. She takes lisinopril, metformin, semaglutide
and simvastatin. Her BMI is 48 kg/m2and has not changed despite lifestyle
advice and a low calorie diet for the last year.

Investigations:
Glycated haemoglobin 55 mmol/mol (20-42)

Which is the most appropriate management?

A. Intensify lifestyle measures and review in 6 months


B. Prescribe orlistat
C. Refer for bariatric surgery
D. Refer for bilateral knee replacements
E. Start insulin therapy
Correct Answer(s): C

Justification for correct answer(s): The patient has severe obesity (BMI > 40
kg/m2) and comorbidities including hypertension and type 2 diabetes, which
puts her at high risk for obesity-related complications. Despite lifestyle
measures and low calorie diet, she has not been able to achieve significant
weight loss. Bariatric surgery is an effective treatment option for obesity in
patients with BMI > 40 kg/m2, or BMI > 35 kg/m2 with comorbidities such as
diabetes and hypertension. The surgery has been shown to improve weight
loss, reduce obesity-related comorbidities, including knee pain, and improve
quality of life.

Clinical guideline [CG189] Published: 27 November 2014 Last updated: 26 July


2023
https://www.nice.org.uk/guidance/cg189/chapter/Recommendations

© Medical Schools Council 2024 – reviewed August 2024


Page 90 of 104
87. A 22 year old soldier steps off a cramped military aircraft following a long flight
from the UK. She suddenly collapses and hits her head on the ground. While
unconscious, she has asynchronous jerking of her limbs for less than 15
seconds. Witnesses say that she looked pale. She regains consciousness
within 1 minute.

What is the most likely cause of her collapse?

A. Cardiac arrhythmia
B. Epilepsy
C. Hypoglycaemia
D. Pulmonary embolism
E. Vasovagal syncope
Correct Answer(s): E

Justification for correct answer(s): The most likely cause of her collapse is
vasovagal syncope. The cramped conditions and fatigue during the flight could
have caused her to experience a vasovagal response, resulting in a temporary
loss of consciousness. The asynchronous jerking of her limbs may have been
due to myoclonus, which can occur during syncope. The pallor may be due to
a transient decrease in blood pressure during the episode. Pulmonary
embolism is a possibility but vasovagal syncope is much more likely.
Epilepsy is a possibility but the duration of shaking would be unusually short.

© Medical Schools Council 2024 – reviewed August 2024


Page 91 of 104
88. A 62 year old man has 2 months of increasing shortness of breath and chest
pain. He is now unable to lie flat. For the past 2 weeks, he has also had a
productive cough which was flecked with blood on two occasions. He had a
myocardial infarction 6 months ago, at which point he stopped smoking.

His temperature is 37.1°C, BP 126/66 mmHg, respiratory rate 24 breaths per


minute and oxygen saturation 93% breathing air.

Investigations:
Chest X-ray: moderate right-sided pleural effusion.

Pleural aspirate protein content 56 g/L.

Which is the most likely underlying diagnosis?

A. Bacterial pneumonia
B. Heart failure
C. Lung cancer
D. Pulmonary embolism
E. Tuberculosis
Correct Answer(s): C

Justification for correct answer(s): The most likely underlying diagnosis is lung
cancer. The high protein content (56 g/L) in the pleural aspirate indicates an
exudative effusion, which more indicative of malignancies like lung cancer.
Heart failure and pulmonary embolism can present with similar symptoms, but
they are less likely given the chest X-ray and pleural aspirate. Bacterial
pneumonia or tuberculosis are also less likely due to a lack of fever.

© Medical Schools Council 2024 – reviewed August 2024


Page 92 of 104
89. A 55 year old man is referred to the vascular outpatient clinic with bilateral
claudication, limiting his walking distance to 10 metres. He is a smoker.

Imaging shows chronic distal aortic and bilateral common iliac occlusive
disease.

Which is the most appropriate surgical intervention?

A. Aortic endarterectomy
B. Aorto-bifemoral bypass graft
C. Aorto-iliac embolectomy
D. Bilateral iliac angioplasty
E. Femoral-to-femoral crossover graft
Correct Answer(s): B

Justification for correct answer(s): Chronic distal aortic and bilateral common iliac
occlusive disease would make aorto-bifemoral bypass graft the most
appropriate surgical intervention. Bypass surgery is offered go people with
severe lifestyle-limiting intermittent claudication when angioplasty has been
unsuccessful, or is unsuitable, and imaging has confirmed that bypass surgery
is appropriate for the person.

Aorto-bifemoral bypass graft involves bypassing the occluded aortic and iliac
vessels with a synthetic graft to restore blood flow to the legs. Other surgical
options like aortic endarterectomy or aorto-iliac embolectomy may not be
suitable for chronic occlusive disease, while bilateral iliac angioplasty and
femoral-to-femoral crossover graft may not be adequate for restoring blood
flow to the entire leg.

Clinical guideline [CG147] Published: 08 August 2012 Last updated: 11


December 2020
https://www.nice.org.uk/guidance/cg147/chapter/recommendations

© Medical Schools Council 2024 – reviewed August 2024


Page 93 of 104
90. A 65 year old man attends his GP for monitoring of hypertension and
ischaemic heart disease. He takes aspirin, atenolol, amlodipine, lisinopril and
simvastatin.

He has marked ankle swelling.

Which drug is the most likely cause of his ankle swelling?

A. Amlodipine
B. Aspirin
C. Atenolol
D. Lisinopril
E. Simvastatin
Correct Answer(s): A

Justification for correct answer(s): Some antihypertensive medications such as


calcium channel blockers like amlodipine, can cause ankle swelling as a side
effect. Lisinopril is an angiotensin-converting enzyme (ACE) inhibitor used to
treat hypertension and heart failure. It is known to cause cough as a side
effect. Statins such as Simvastatin and beta blockers such as Atenolol are not
known to cause ankle swelling as a side effect.

© Medical Schools Council 2024 – reviewed August 2024


Page 94 of 104
91. An 85 year old woman was admitted with a stroke three weeks ago. She has
urinary incontinence and a long-term urinary catheter in situ. She
takes clopidogrel and ramipril.

Her temperature is 36.8°C, pulse rate 85 bpm and BP 134/74 mmHg.A


catheter specimen of urine shows >105CFU/mL, mixed growth.

Which is the most appropriate management?

A. No change in treatment
B. Remove urinary catheter
C. Request antibiotic sensitivities
D. Start oral ciprofloxacin
E. Start oral trimethoprim
Correct Answer(s): A

Justification for correct answer(s): Colonisation of urinary catheters with a mixed


growth of bacteria is very common, does not usually cause symptoms and is
not an infection that needs treatment with antibiotics in most patients. It would
be inappropriate to remove the catheter, in this patient with a previous history
of incontinence who in addition has had a stroke and will be at increased risk
of pressure area damage. There are no indications for antibiotics. Indwelling
urinary catheters quickly become colonised with microorganisms after
insertion. These microorganisms produce proteins and facilitate the formation
of biofilms. These biofilms often make it impossible to eradicate the bacteria
with antibiotics.

© Medical Schools Council 2024 – reviewed August 2024


Page 95 of 104
92. A 28 year old man has a headache, intermittent fever, sore throat and
diarrhoea.

His temperature is 37.7°C. His fauces are red and there are two small
aphthous ulcers on his left buccal mucosa. He also has a maculopapular
erythematous rash on his upper trunk, red hands and folliculitis on his chest.
His liver and spleen are just palpable and he has mild neck stiffness.

Investigations:
Haemoglobin 135 g/L (130–175)
White cell count 3.3 x 109/L (3.0–10.0)
Platelets 84 x 109/L (150–400)

Which investigation is most likely to lead to a diagnosis?

A. First catch urine microscopy


B. Glandular fever screening test
C. HIV serology
D. Serum antinuclear antibodies
E. Serum toxoplasma gondii IgM antibody titre
Correct Answer(s): C

Justification for correct answer(s): The presentation suggests an infection, which


is affecting a number of different body regions and systems. The most specific
information is the presence of a rash with folliculitis on the chest, which is a
prominent feature in late stage HIV infection. Additionally, HIV can explain all
of the symptoms, hence the correct answer is HIV serology.

© Medical Schools Council 2024 – reviewed August 2024


Page 96 of 104
93. A 79 year old woman has 6 months of increasing breathlessness on exertion.

Her pulse is 72 bpm, irregularly irregular, and BP 118/72 mmHg. She has a
diastolic murmur best heard at the apex in expiration.

Which is the most likely cause of her murmur?

A. Aortic regurgitation
B. Aortic stenosis
C. Hypertrophic cardiomyopathy
D. Mitral regurgitation
E. Mitral stenosis
Correct Answer(s): E

Justification for correct answer(s): Based on the location and timing of the
murmur, the most likely cause is Mitral stenosis.

© Medical Schools Council 2024 – reviewed August 2024


Page 97 of 104
94. A 78 year old woman is found dead at home. At autopsy, the pathologist finds
bilateral pneumonia and meningitis.

Microscopy of a meningeal swab shows Gram-positive cocci arranged in


pairs.

Which is the most likely causative organism?

A. Candida albicans
B. Neisseria meningitidis
C. Pseudomonas aeruginosa
D. Staphylococcus aureus
E. Streptococcus pneumoniae
Correct Answer(s): E

Justification for correct answer(s): The most likely causative organism in this case
is Streptococcus pneumoniae. Streptococcus pneumoniae is a Gram-positive
coccus that can cause pneumonia and meningitis, particularly in the elderly.
The presence of bilateral pneumonia and meningitis, as well as the Gram-
positive cocci seen on microscopy, are consistent with this diagnosis. Of the
other possible answers, Neisseria meningitidis, Candida albicans and
Pseudomonas aeruginosa are not Gram positive cocci. Staph aureus tends to
form clusters rather than being arranged in pairs.

© Medical Schools Council 2024 – reviewed August 2024


Page 98 of 104
95. A 40 year old man was admitted with central crushing chest pain.

He has pale cream coloured nodules on both elbows and medial aspects of
his upper eyelids.

ECG on admission showed ST elevation and T wave inversion.

He deteriorated and died. A post mortem examination is performed and


shows very severe narrowing of the anterior descending branch of the left
coronary artery.

Which is the most likely causative mechanism?

A. Atheroma
B. Arterial dissection
C. Malignant deposit
D. Thrombosis
E. Vasculitis
Correct Answer(s): A

Justification for correct answer(s): The most likely causative mechanism of the
severe narrowing of the anterior descending branch of the left coronary artery
is atheroma, given the patient’s clinical presentation of central crushing chest
pain, ST elevation and T wave inversion on ECG, and subsequent deterioration
and death and the post mortem findings. The presence of pale cream coloured
nodules on both elbows and medial aspects of the upper eyelids suggests the
possibility of xanthomas, which are associated with hyperlipidemia and can be
seen in patients with atheromatous plaques. However, thrombosis on the
atheroma is likely to cause the acute presentation and fatal outcome. Arterial
dissection, malignant deposits, and vasculitis are less likely causes in this
clinical scenario.

© Medical Schools Council 2024 – reviewed August 2024


Page 99 of 104
96. A 55 year old man attends the GP surgery concerned that he may be a carrier
of cystic fibrosis. The condition has just been diagnosed in his 5 year old
grandson. He has heard that this is an inherited condition, but no one else in
his family has the illness.

What is the likelihood that the grandfather is a carrier?

A. 1 in 2
B. 1 in 4
C. 1 in 8
D. 1 in 16
E. 1 in 25
Correct Answer(s): A

Justification for correct answer(s): Cystic fibrosis is an autosomal recessive


disorder, which means that an individual must inherit two copies of the
mutated gene (one from each parent) to develop the disease. If the grandchild
is affected, both of his parents must be carriers. The parent of the 5 year old
must have received the recessive gene from one of their parents. There is a
50:50 chance that this was the grandfather and 50:50 that it was the
grandmother. This means that the likelihood that the grandfather is a carrier is
1 in 2.

© Medical Schools Council 2024 – reviewed August 2024


Page 100 of 104
97. A 79 year old woman has been repeatedly found wandering at night by her
neighbours. This has progressively worsened over 6 months. She is
independent in her activities of daily living, although her family do her
shopping. She was previously well.

What aspect of cognition is likely to show the greatest impairment?

A. Attention
B. Concentration
C. Praxis
D. Registration of information
E. Short-term memory
Correct Answer(s): E

Justification for correct answer(s): The scenario describes a patient with


symptoms of dementia, and impairment of short-term memory is a
characteristic feature of dementia. The patient’s wandering behaviour may be
due to disorientation caused by forgetfulness or confusion. Attention,
concentration, and praxis may also be affected in dementia, but short-term
memory is often the most severely impaired.

© Medical Schools Council 2024 – reviewed August 2024


Page 101 of 104
98. A 52 year old woman has increased urinary frequency, urgency and urge
incontinence. She has multiple sclerosis, which affects her walking. A
midstream urine sample shows no cells and is sterile on culture. A bladder
scan shows a residual volume of 300 mL. Urodynamic assessment shows
that she has a neuropathic bladder.

Which is the most appropriate management?

A. α-Adrenoceptor blocker
B. Anticholinergic drug
C. Indwelling urethral catheter
D. Intermittent self catheterisation
E. Suprapubic catheter
Correct Answer(s): D

Justification for correct answer(s): The most appropriate management in this case
of a patient with a neuropathic bladder due to multiple sclerosis would be
intermittent self-catheterization. Drug interventions are unlikely to be of
benefit. Indwelling urethral catheter or suprapubic catheter are to be avoided
due to increase infection risk.

© Medical Schools Council 2024 – reviewed August 2024


Page 102 of 104
99. A 43 year old woman is admitted with acute right upper quadrant pain, which
radiates to her right shoulder.

Her temperature is 38.6°C and respiratory rate 20 breaths per minute. She is
tender to palpation in the right upper quadrant but has no rebound
tenderness.

Investigations:
Haemoglobin 132 g/L (115–150)
White cell count 13 x 109/L (3.8–10.0)
Platelets 340 x 109/L (150–400)
Bilirubin 30 µmol/L (<17)
Alanine aminotransferase (ALT) 80 IU/L (10–50)
Alkaline phosphatase 306 IU/L (25–115)

Which is the next most appropriate radiological test?

A. Abdominal X-ray
B. CT scan of abdomen
C. Erect chest X-ray
D. MR scan of abdomen
E. Ultrasound scan of abdomen
Correct Answer(s): E

Justification for correct answer(s): The most appropriate radiological test for
suspected acute cholecystitis is an ultrasound scan of the abdomen.

© Medical Schools Council 2024 – reviewed August 2024


Page 103 of 104
100. An 84 year old man develops profuse diarrhoea whilst in hospital. An outbreak
of Clostridioides (Clostridium) difficile has occurred in his ward.

Which feature of this organism makes it particularly difficult to destroy?

A. Motility
B. Outer capsule
C. Rapid mutation
D. Spore formation
E. Surface adherence
Correct Answer(s): D

Justification for correct answer(s): The feature of Clostridioides difficile that


makes it particularly difficult to destroy is spore formation. C. difficile spores
are resistant to many environmental stresses, including heat, disinfectants,
and antibiotics, which makes them particularly difficult to eliminate. These
spores can persist on surfaces for months, making them a significant source
of transmission in healthcare settings.

© Medical Schools Council 2024 – reviewed August 2024


Page 104 of 104

You might also like